SlideShare a Scribd company logo
1 of 112
Hypo/hyperglycemic coma
Hypoglycemic coma DKA HO
Etiology Access of insulin in organism
1) related to diabetes:
missed, delayed or inadequate
meal;
unexpected or unusual exercise;
alcohol;
errors in oral hypoglycemic
agents or insulin
dose/schedule/administration;
2)related to people
with/without diabetes:
chronic kidney disease IV-V
stage;
liver failure;
counter regulatory failure:
= adrenal failure;
= hypopituitarism;
= malabsorbtion;
= insulinoma.
Lack of insulin
1)newly diagnosed
diabetes (presenting
manifestation);
2)inadequate
administration of
exogenous insulin;
3)increased
requirements for
insulin caused by the
presence of an
underlying stressful
condition:
an intercurrent
infection ;
a vascular disorder
(myocardial infarction,
stroke);
an endocrine disorder ;
trauma; pregnancy;
surgery.
Dehydration of
different
etiology
Hypoglycemic coma DKA HO
Onset Rapid Gradual Gradual
Clinical
signs
1) adrenergic symptoms :
= sweating;
= nervousness;
= tremulousness;
= faintness;
= anxiety;
= palpitation;
= hunger;
2) cerebral nervous system
manifestations:
= confusion;
= dizziness;
= inappropriate behavior (which
can be mistaken for inebriation);
= visual disturbances;
= headache;
= seizures;
= loss of fine motor skills;
= loss of consciousness.
- Polydipsia,
polyuria and
weakness are the
most common
presenting
complaints.
- Anorexia, nausea,
vomiting, and
abdominal pain
may be present and
mimic an
abdominal
emergency.
- Ileus and gastric
dilatation may
occur and
predispose to
aspiration.
-Polyuria, polydipsia,
weight loss, weakness
- progressive
changes in state of
consciousness
from mental
cloudiness to coma
- underlying
conditions (such as
cerebrovascular
accident and
subdural
hematoma)
- Seizures occur in 5
% of patients and
may be either focal
or generalized
Hypoglycemic coma DKA HO
Physical
examination
- The skin is cold,
moist.
-Full pulse.
- Normal or raised
blood pressure.
- Shallow or normal
breathing.
- Hyperreflexia can
be elicited.
- Hypoglycemic
coma is commonly
associated with
abnormally low
body temperature
- Patient may be
unconsciousness.
- The skin is dry
- Hypothermia is
common in DKA. A
fever should be taken
as strong evidence of
infection.
- Tachycardia
frequently is present
- Blood pressure is
usually normal unless
profound dehydration
is present.
- Hyperpnoea or
Kussmaul respiration
- acetone may be
detected on the
breath (musty (fruity)
odor to the breath).
- Hyporeflexia .
- Severe dehydration is
invariably present.
- Various neurologic
deficits (such as
coma, transient
hemiparesis,
hyperreflexia, and
generalized areflexia)
are commonly
present.
- Altered states of
consciousness from
lethargy to coma are
observed.
- Findings associated
with coexisting
medical problems
(e.g., renal disease,
cardiovascular
disease) may be
evident
Hypoglycemic
coma
DKA HO
Laboratory
features
Low level of
blood glucose
- hyperglycemia;
- ketonemia;
- metabolic
acidosis (plasma
pH and
bicarbonates are
decreased,
- potassium
disturbances
- Extreme hyperglycemia (blood
glucose levels from 30 mmoll/l
and over are common.
- A markedly elevated serum
osmolality is present, usually in
excess of 350 mOsm/l. (Normal =
290 mOsm/l).
- Serum ketones are usually not
detectable, and patients are not
acidic.
- Serum sodium may be high
- Serum potassium levels may be
high (secondary to the effects of
hyperosmolality as it draws
potassium from the cells), normal,
or low (from marked urinary
losses from the osmotic diuresis).
But potassium deficiency exists.
• A 23 y.o. woman who suffers from insulin-
dependent diabetes was admitted to the acute
care department with mental confusion,
inadequate anxious behaviour, hyperhidrosis,
excessive salivation, tachycardia.
• What examination will be a primary task?
A. Clinical blood analysis
B. Plasma electrolytes test
C. Gaseous composition of arterial blood
D. Blood test for sugar
E. Blood urea and creatinine test
• A 23 y.o. woman who suffers from insulin-
dependent diabetes was admitted to the acute
care department with mental confusion,
inadequate anxious behaviour, hyperhidrosis,
excessive salivation, tachycardia.
• What examination will be a primary task?
A. Clinical blood analysis
B. Plasma electrolytes test
C. Gaseous composition of arterial blood
D. Blood test for sugar
E. Blood urea and creatinine test
• An unconscious patient presents with moist skin,
shallow breathing. There are signs of previous
injection on the shoulders and hips. BP- 110/70
mm Hg. Tonus of skeletal muscles and reflexes
are increased. Cramps of muscles of the
extremities are seen.
• What is the most likely disorder?
A. Hyperglycemic coma
B. Hyperosmolar coma
C. Hypoglycemic coma
D. Hyperlactacidotic coma
E. Stroke
• An unconscious patient presents with moist skin,
shallow breathing. There are signs of previous
injection on the shoulders and hips. BP- 110/70
mm Hg. Tonus of skeletal muscles and reflexes
are increased. Cramps of muscles of the
extremities are seen.
• What is the most likely disorder?
A. Hyperglycemic coma
B. Hyperosmolar coma
C. Hypoglycemic coma
D. Hyperlactacidotic coma
E. Stroke
• A 9 year old boy had acute respiratory viral infection.
After it there appeared polydipsia, polyuria,
weakness, nausea. Examination revealed the following
symptoms: mental confusion, dry skin, soft eyeballs,
Kussmaul’s respiration, acetone smell from the
mouth, muffled heart sounds, soft and painless
abdomen. Blood sugar was 19 millimole/l.
• What acute condition is it?
A. Acute renal insufficiency
B. Hyperosmolar coma
C. Cerebral coma
D. Hepatic coma
E. Ketoacidotic coma
• A 9 year old boy had acute respiratory viral infection.
After it there appeared polydipsia, polyuria, weakness,
nausea. Examination revealed the following symptoms:
mental confusion, dry skin, soft eyeballs, Kussmaul’s
respiration, acetone smell from the mouth, muffled
heart sounds, soft and painless abdomen. Blood sugar
was 19 millimole/l.
• What acute condition is it?
A. Acute renal insufficiency
B. Hyperosmolar coma
C. Cerebral coma
D. Hepatic coma
E. Ketoacidic coma
• A 3-year-old child has been diagnosed with type I
diabetes mellitus, hyperosmolar coma. The
laboratory confirmed the diagnosis.
• Which laboratory findings are characteristic for
such condition?
A. High hyperglycemia without ketonemia
B. Hyperglycemia and ketonemia
C. Hyperglycemia and glucosuria
D. Hyperglycemia and ketonuria
E. Hyperglycemia and high indicators of acid-base
balance
• A 3-year-old child has been diagnosed with type I
diabetes mellitus, hyperosmolar coma. The
laboratory confirmed the diagnosis.
• Which laboratory findings are characteristic for
such condition?
A. High hyperglycemia without ketonemia
B. Hyperglycemia and ketonemia
C. Hyperglycemia and glucosuria
D. Hyperglycemia and ketonuria
E. Hyperglycemia and high indicators of acid-base
balance
• An 8-year-old child with a 3-year-long history of
diabetes was hospitalized in hyperglycemic
coma.
• Specify the initial dose of insulin to be
administered:
A. 0,1-0,2 U/kg of body weight per hour
B. A. 0,05 U/kg of body weight per hour
C. B. 0,2-0,3 U/kg of body weight per hour
D. 0,3-0,4 U/kg of body weight per hour
E. 0,4-0,5 U/kg of body weight per hour
• An 8-year-old child with a 3-year-long history of
diabetes was hospitalized in hyperglycemic
coma.
• Specify the initial dose of insulin to be
administered:
A. 0,1-0,2 U/kg of body weight per hour
B. A. 0,05 U/kg of body weight per hour
C. B. 0,2-0,3 U/kg of body weight per hour
D. 0,3-0,4 U/kg of body weight per hour
E. 0,4-0,5 U/kg of body weight per hour
• During examination a patient is unconscious, his skin is
dry and hot, face hyperemia is present. The patient
has Kussmaul’s respiration, there is also smell of
acetone in the air. Symptoms of peritoneum irritation
are positive. Blood sugar is 33 mmol/l.
• What emergency actions should be taken?
A. Intravenous infusion of short-acting insulin
B. Intravenous infusion of glucose along with insulin
C. Introduction of long-acting insulin
D. Intravenous infusion of neohaemodesum along with
glutamic acid
E. Intravenous infusion of sodium chloride saline
• During examination a patient is unconsci-ous, his skin is
dry and hot, face hyperemia is present. The patient has
Kussmaul’s respiration, there is also smell of acetone in
the air. Symptoms of peritoneum irritation are positive.
Blood sugar is 33 mmol/l.
• What emergency actions should be taken?
A. Intravenous infusion of short-acting insulin
B. Intravenous infusion of glucose along with insulin
C. Introduction of long-acting insulin
D. Intravenous infusion of neohaemodesum along with
glutamic acid
E. Intravenous infusion of sodium chloride saline
• A 23-year-old woman presents with diabetes mellitus type 1. She
complains of weakness, headache, nausea, and vomiting.
Objectively: temperature is 37,6°C, heart rate is 98/min., BP is
95/65 mm Hg, respiration rate is 32/min., loud. Smell of acetone is
detected, heart sounds are muffled, pulse is rhythmic. The stomach
is sensitive in the epigastrium. Costovertebral angle tenderness
(Murphy’s punch sign) is present. Blood glucose is 28,5 mmol/l;
blood leukocytes - 16, 5 • 109/l. In urine: acetone ++, leukocytes -
25-40 in the vision field. Blood pH is 7,1.
• What correction method would be the most advisable?
A. Infusion of normal saline + insulinotherapy in small portions*
B. Infusion of 5% glucose + insulinotherapy in large portions
C. Infusion of 2,5% sodium bicarbonate + insulinotherapy
D. Intravenous administration of antibiotics + insulinotherapy
E. Infusion of dextran solutions + insuli-notherapy
• A 23-year-old woman presents with diabetes mellitus type 1. She
complains of weakness, headache, nausea, and vomiting.
Objectively: temperature is 37,6°C, heart rate is 98/min., BP is
95/65 mm Hg, respiration rate is 32/min., loud. Smell of acetone is
detected, heart sounds are muffled, pulse is rhythmic. The stomach
is sensitive in the epigastrium. Costovertebral angle tenderness
(Murphy’s punch sign) is present. Blood glucose is 28,5 mmol/l;
blood leukocytes - 16, 5 • 109/l. In urine: acetone ++, leukocytes -
25-40 in the vision field. Blood pH is 7,1.
• What correction method would be the most advisable?
A. Infusion of normal saline + insulinotherapy in small portions
B. Infusion of 5% glucose + insulinotherapy in large portions
C. Infusion of 2,5% sodium bicarbonate + insulinotherapy
D. Intravenous administration of antibiotics + insulinotherapy
E. Infusion of dextran solutions + insuli-notherapy
• A 23-year-old woman after stress has developed thirst,
polydipsia, polyuria, weight loss, increasing fatigue. Later
she developed nausea and somnolence, lost
consciousness and was hospitalised. Glycemia is 27
mmol/l, acetone in urine is sharply positive. Treatment
for ketoacidotic coma was initiated.
• When would it be advisable to start preventive treatment
of hypoglycemia by introduction of 5% glucose solution?
A. After glycemia rate drops to 13-14 mmol/l
B. 2 hours after beginning of insulinotherapy
C. When patient becomes conscious
D. After glycemia rate becomes normal
E. If glycemia decreases with the rate over 5 mmol/l per
нour
• A 23-year-old woman after stress has developed thirst,
polydipsia, polyuria, weight loss, increasing fatigue. Later
she developed nausea and somnolence, lost consciousness
and was hospitalised. Glycemia is 27 mmol/l, acetone in
urine is sharply positive. Treatment for ketoacidotic coma
was initiated.
• When would it be advisable to start preventive treatment
of hypoglycemia by introduction of 5% glucose solution?
A. After glycemia rate drops to 13-14 mmol/l
B. 2 hours after beginning of insulinotherapy
C. When patient becomes conscious
D. After glycemia rate becomes normal
E. If glycemia decreases with the rate over 5 mmol/l per
hour
• A 72-year-old woman suffers from diabetes mellitus
type 2, concomitant diseases are stage 2 hypertension
and stage 2B heart failure. She takes metformin.
Hypertensic crisis had occurred the day before, after
which the patient developed extreme weakness,
myalgias, thirst, dry mouth, polyuria. BP is 140/95 mm
Hg, heart rate is 98/min., no edemas or smell of
acetone detected.
• What measures should be taken to prevent
development of comatose state in the patient?
A. Stop metformin, prescribe short-acting insulin
B. Double the dosage of metformin
C. Apply hypotonic solution of sodium chloride
D. Additionally prescribe long-acting insulin
E. Prescribe glibenclamid
• A 72-year-old woman suffers from diabetes mellitus
type 2, concomitant diseases are stage 2 hypertension
and stage 2B heart failure. She takes metformin.
Hypertensic crisis had occurred the day before, after
which the patient developed extreme weakness,
myalgias, thirst, dry mouth, polyuria. BP is 140/95 mm
Hg, heart rate is 98/min., no edemas or smell of
acetone detected.
• What measures should be taken to prevent
development of comatose state in the patient?
A. Stop metformin, prescribe short-acting insulin
B. Double the dosage of metformin
C. Apply hypotonic solution of sodium chloride
D. Additionally prescribe long-acting insulin
E. Prescribe glibenclamid
Step
• 51. A 27-year-old nurse comes to the emergency department because of
nervousness, dizziness, palpitations, and excess perspiration for the past
3 hours. She has had similar episodes over the past 6 months. The
symptoms improve following ingestion of orange juice or soft drinks. She
says that she has had a great deal of stress. She has been drinking two
alcoholic beverages daily for the past month; before this time, she seldom
drank alcohol. Examination shows no abnormalities. Her serum glucose
concentration is 30 mg/dL. Intravenous glucose is administered, and the
patient's symptoms improve.
• Which of the following is the most appropriate next step in diagnosis?
A. Liver tests
B. Measurement of serum proinsulin and insulin antibodies
C. Measurement of serum cortisol and ACTH concentrations
D. Measurement of serum growth hormone and plasma somatomedin-C
concentrations
E. Measurement of serum insulin and C-peptide concentrations
• 51. A 27-year-old nurse comes to the emergency department because of
nervousness, dizziness, palpitations, and excess perspiration for the past
3 hours. She has had similar episodes over the past 6 months. The
symptoms improve following ingestion of orange juice or soft drinks. She
says that she has had a great deal of stress. She has been drinking two
alcoholic beverages daily for the past month; before this time, she seldom
drank alcohol. Examination shows no abnormalities. Her serum glucose
concentration is 30 mg/dL. Intravenous glucose is administered, and the
patient's symptoms improve.
• Which of the following is the most appropriate next step in diagnosis?
A. Liver tests
B. Measurement of serum proinsulin and insulin antibodies
C. Measurement of serum cortisol and ACTH concentrations
D. Measurement of serum growth hormone and plasma somatomedin-C
concentrations
E. Measurement of serum insulin and C-peptide concentrations
• 54. Three days after hospitalization for diabetic ketoacidosis, an 87-year-
old woman refuses insulin injections. She says that her medical condition
has declined so much that she no longer wishes to go on living; she is
nearly blind and will likely require bilateral leg amputations. She reports
that she has always been an active person and does not see how her life
will be of value anymore. She has no family and most of her friends are
sick or deceased. On mental status examination, she is alert and
cooperative. She accurately describes her medical history and
understands the consequences of refusing insulin. There is no evidence of
depression. She dismisses any attempts by the physician to change her
mind, saying that the physician is too young to understand her situation.
She says, "I know I will die, and this is what I want."
• Which of the following is the most appropriate next step in management?
A. Discharge the patient after she has signed an "against medical advice"
form
B. Seek a court order to appoint a legal guardian
C. Offer insulin but allow the patient to refuse it
D. Admit to the psychiatric unit
E. Administer insulin against the patient's wishes
• 54. Three days after hospitalization for diabetic ketoacidosis, an 87-year-
old woman refuses insulin injections. She says that her medical condition
has declined so much that she no longer wishes to go on living; she is
nearly blind and will likely require bilateral leg amputations. She reports
that she has always been an active person and does not see how her life
will be of value anymore. She has no family and most of her friends are
sick or deceased. On mental status examination, she is alert and
cooperative. She accurately describes her medical history and
understands the consequences of refusing insulin. There is no evidence of
depression. She dismisses any attempts by the physician to change her
mind, saying that the physician is too young to understand her situation.
She says, "I know I will die, and this is what I want."
• Which of the following is the most appropriate next step in management?
A. Discharge the patient after she has signed an "against medical advice"
form
B. Seek a court order to appoint a legal guardian
C. Offer insulin but allow the patient to refuse it
D. Admit to the psychiatric unit
E. Administer insulin against the patient's wishes
• A previously healthy 18-year-old man is brought to the emergency
department because of abdominal pain and nausea for 6 hours. He
has had decreased appetite for the past week. He takes no
medications. He drinks one to two beers daily and occasionally
more on weekends. He does not use illicit drugs. His temperature is
37.8°C (100°F), pulse is 120/min, respirations are 24/min, and
blood pressure is 105/60 mm Hg. Abdominal examination shows
diffuse tenderness with no guarding or rebound. Bowel sounds are
normal. Laboratory studies show:
• Serum Na+ 135 mEq/L Cl− 98 mEq/L K+ 3.8 mEq/L HCO3− 16 mEq/L
Glucose 360 mg/dL Ketones present Urine ketones present
• Arterial blood gas analysis on room air shows a pH of 7.30. Which of
the following is the most likely diagnosis?
A. Acute appendicitis
B. Acute pancreatitis
C. Alcoholic ketoacidosis
D. Diabetic ketoacidosis
E. Lactic acidosis
• A previously healthy 18-year-old man is brought to the emergency
department because of abdominal pain and nausea for 6 hours. He
has had decreased appetite for the past week. He takes no
medications. He drinks one to two beers daily and occasionally
more on weekends. He does not use illicit drugs. His temperature is
37.8°C (100°F), pulse is 120/min, respirations are 24/min, and blood
pressure is 105/60 mm Hg. Abdominal examination shows diffuse
tenderness with no guarding or rebound. Bowel sounds are normal.
Laboratory studies show:
• Serum Na+ 135 mEq/L Cl− 98 mEq/L K+ 3.8 mEq/L HCO3− 16 mEq/L
Glucose 360 mg/dL Ketones present Urine ketones present
• Arterial blood gas analysis on room air shows a pH of 7.30. Which of
the following is the most likely diagnosis?
A. Acute appendicitis
B. Acute pancreatitis
C. Alcoholic ketoacidosis
D. Diabetic ketoacidosis
E. Lactic acidosis
Acute adrenal failure
Predisposing factors  Diagnostic criteria Laboratory findings
• sudden withdrawal of
corticosteroid
hormone therapy in
patients with chronic
insufficiency and non-
endocrine disorders;
• stress or infection in
patints with Adisson’s
disease (especially with
septicemia, trauma,
surgery, prolonged
fasting, salt loss due to
excessive sweating
during hot weather);
• thrombosis or
embolism of primary
affected adrenals
(Waterhouse-
Friderichsen
syndrome).
• An adrenal crisis is
characterized by profound
asthenia, cardiovascular,
gastrointestinal, neuro-
psychyatric disorders.
• Cardiovascular disorders
include hypotension,
cyanosis, dyspnea.
• Hypotension and peripheral
vascular collapse is result of
gastrointestinal
disturbances (nausea,
vomiting diarrhea) also can
lead to renal shutdown with
azotemia.
• Body temperature may be
subnormal, through severe
hyperthermia due to
infection is often seen.
• electrolyte
disturbances: a low
serum sodium level
and a high serum
potassium level
• hypoglycemia
• decreased cortisol
level
• ACTH increased in
primary and
decreased in
secondary adrenal
insufficiency
• A 34 y.o. patient has been suffering from pulmonary
tuberculosis for 7 years; he complains of muscle weakness,
weight loss, diarrhea, frequent urination. Objectively:
hyperpigmentation of skin, gums, internal surface of
cheeks. AP- 90/58 mm Hg. Blood count: RBC- 3, 1 ∗ 1012/L,
Hb- 95 g/L, C.I.- 0,92; leukocytes - 9, 4 ∗9/L, eosinophils - 7,
segmentonuclear leukocytes - 45, stab neutrophils - 1,
lymphocytes - 40, monocytes - 7, Na+- 125 mmol/L, K+- 7,3
mmol/L.
• What is the preliminary diagnosis?
A. Pheochromocytoma
B. Primary hyperaldosteronism
C. Congenital adrenocortical hyperplasia
D. Diabetes insipidus
E. Primary adrenocortical insufficiency
• A 34 y.o. patient has been suffering from pulmonary
tuberculosis for 7 years; he complains of muscle weakness,
weight loss, diarrhea, frequent urination. Objectively:
hyperpigmentation of skin, gums, internal surface of
cheeks. AP- 90/58 mm Hg. Blood count: RBC- 3, 1 ∗ 1012/L,
Hb- 95 g/L, C.I.- 0,92; leukocytes - 9, 4 ∗9/L, eosinophils - 7,
segmentonuclear leukocytes - 45, stab neutrophils - 1,
lymphocytes - 40, monocytes - 7, Na+- 125 mmol/L, K+- 7,3
mmol/L.
• What is the preliminary diagnosis?
A. Pheochromocytoma
B. Primary hyperaldosteronism
C. Congenital adrenocortical hyperplasia
D. Diabetes insipidus
E. Primary adrenocortical insufficiency
• After having the flu, a 39-year-old male patient with a
history of Addison’s disease developed a condition
manifested by weakness, depression, nausea, vomi-
ting, diarrhea, hypoglycemia. AP- 75/50 mm Hg. Blood
test results: low corticosterone and cortisol, 13-
oxycorticosteroids, 17-oxycorticosteroids levels.
• What condition developed in the patient?
A. Acute adrenal insufficiency
B. Acute gastritis
C. Acute enterocolitis
D. Collapse
E. Diabetes mellitus
• After having the flu, a 39-year-old male patient with a
history of Addison’s disease developed a condition
manifested by weakness, depression, nausea, vomi-
ting, diarrhea, hypoglycemia. AP- 75/50 mm Hg. Blood
test results: low corti-costerone and cortisol, 13-
oxycorticosteroids, 17-oxycorticosteroids levels.
• What condition developed in the patient?
A. Acute adrenal insufficiency
B. Acute gastritis
C. Acute enterocolitis
D. Collapse
E. Diabetes mellitus
• A 41-year-old patient with Addison’s disease had
influenza. After that he developed adynamia,
depression, nausea, vomiting, diarrhea and
hypoglycemia. BP is 75/50 mm Hg. Blood test:
decreased content of corticosterone, hydrocortisone,
13-oxycorticosteroids, 17-oxycorticosteroids.
• What condition has developed in the patient?
A. Acute adrenal gland insufficiency
B. Acute gastritis
C. Acute enterocolitis
D. Collapse
E. Diabetes mellitus
• A 41-year-old patient with Addi-son’s disease had
influenza. After that he developed adynamia,
depression, nausea, vomiting, diarrhea and
hypoglycemia. BP is 75/50 mm Hg. Blood test:
decreased content of corticosterone, hydrocortisone,
13-oxycorticosteroids, 17-oxycorticosteroids.
• What condition has developed in the patient?
A. Acute adrenal gland insufficiency
B. Acute gastritis
C. Acute enterocolitis
D. Collapse
E. Diabetes mellitus
• A 43-year-old female patient was delivered to the hospital
in grave condition. She has a history of Addison’s disease.
The patient had been regularly taking prednisolone but a
week before she stopped taking this drug. Objectively:
sopor, skin and visible mucous membranes are pigmented,
skin and muscle turgor is decreased. Heart sounds are
muffled, rapid. AP- 60/40 mm Hg, heart rate - 96/min. In
blood: Na - 120 mi-llimole/l, K - 5,8 millimole/l.
• Development of this complication is primarily caused by
the deficit of the following hormone:
A. Cortisol
B. Corticotropin (ACTH)
C. Adrenaline
D. Noradrenaline
E. Adrostendion
• A 43-year-old female patient was delivered to the hospital
in grave condi-tion. She has a history of Addison’s di-sease.
The patient had been regularly taking prednisolone but a
week before she stopped taking this drug. Objecti-vely:
sopor, skin and visible mucous membranes are pigmented,
skin and muscle turgor is decreased. Heart sounds are
muffled, rapid. AP- 60/40 mm Hg, heart rate - 96/min. In
blood: Na - 120 mi-llimole/l, K - 5,8 millimole/l.
• Development of this complication is primarily caused by
the deficit of the following hormone:
A. Cortisol
B. Corticotropin (ACTH)
C. Adrenaline
D. Noradrenaline
E. Adrostendion
Pheochromocytoma
Etiology  Diagnostic criteria Laboratory findings
is unknown Symptoms “The 5 P’s”
• Pressure increase
(hypertension )
• Palpitation (tachycardia)
• Perspiration
• Pain (abrupt onset of
throbbing headache, chest
(angina), abdominal pain)
• Pallor (due to
vasoconstriction) cold and
clammy skin
• An increased 3-h (24-h) urinary
excretion of epinephrine,
norepinephrine and their metabolic
products (VMA or metanephrines).
• Plasma free metanephrine is up to 99%
sensitive. Imaging tests to localize
tumor:
• CT scanning and MRI of the chest and
abdomen with and without contrast.
• Radiopharmaceuticals with nuclear
imaging techniques can help to localize
pheochromocytomas when the lesion
is large enough to be obvious on CT or
MRI. 123I-metaiodobenzylguanidine
(MIBG) is the most used compound for
this investigation. Normal adrenal
tissue rarely picks up this isotope, but
90% of pheochromocytomas do.
• A 32-year-old woman complains of dizziness, headache,
palpitation, tremor. For the last several months she has
been under outpatient monitoring for increased arterial
pressure. Recently such attacks have become more
frequent and severe. Objectively: skin is covered with
clammy sweat, tremor of the extremities is present. Heart
rate - 110/min, BP - 220/140 mm Hg. Heart sounds are
weakened. In blood: WBCs - 9, 8 · 109/l, ESR - 22 mm/h.
Blood glucose - 9,8 millimole/l.
• What disease is the most likely cause of this crisis?
A. Pheochromocytoma
B. Essential hypertension
C. Preeclampsia
D. Primary hyperaldosteronism
E. Diabetic glomerulosclerosis
• A 32-year-old woman complains of dizziness, headache,
palpitation, tremor. For the last several months she has been
under outpatient monitoring for increased arterial pressure.
Recently such attacks have become more frequent and
severe. Objectively: skin is covered with clammy sweat,
tremor of the extremities is present. Heart rate - 110/min, BP
- 220/140 mm Hg. Heart sounds are weakened. In blood:
WBCs - 9, 8 · 109/l, ESR - 22 mm/h. Blood glucose - 9,8
millimole/l.
• What disease is the most likely cause of this crisis?
A. Pheochromocytoma
B. Essential hypertension
C. Preeclampsia
D. Primary hyperaldosteronism
E. Diabetic glomerulosclerosis
• A 51-year-old woman complains of headache,
trembling, paresthesiae, palpitations, increased blood
pressure up to 280/160 mm Hg. The day before she
experienced exhausting headache, vascular pulsation,
palpitations, asphyxia, stomachache, unbearable fear
of coming death. The patient paled and broke out in
cold sweat. In urine there is increased content of
vanillylmandelic acid.
• What disease causes such clinical presentation in the
patient?
A. Pheochromocytoma
B. Conn’s syndrome (primary hyperaldosteronism)
C. Cushing’s syndrome
D. Primary hypertension
E. Cushing’s disease
• A 51-year-old woman complains of headache,
trembling, paresthesiae, palpitations, increased blood
pressure up to 280/160 mm Hg. The day before she
experienced exhausting headache, vascular pulsation,
palpitations, asphyxia, stomachache, unbearable fear
of coming death. The patient paled and broke out in
cold sweat. In urine there is increased content of
vanillylmandelic acid.
• What disease causes such clinical presentation in the
patient?
A. Pheochromocytoma
B. Conn’s syndrome (primary hyperaldosteronism)
C. Cushing’s syndrome
D. Primary hypertension
E. Cushing’s disease
• A 38 y.o. woman suffers from paroxysmal AP rises up to
240/120 mm Hg accompanied by nausea, vomiting,
tachycardia, excessive sweating. During the onset blood is
hyperglycemic. After the onset there is voluminous
urination. Kidneys sonography revealed accessory mass
bordering upon the upper pole of the right kidney;
presumably it belongs to the adrenal gland.
• What laboratory test will allow making a more precise
diagnosis?
A. Estimation of catecholamine and vanillyl-mandelic acid
excretion with urine
B. Estimation of insulin and C-peptide content in blood
C. Estimation of glomerular filtration rate
D. Estimation of thyroxin and thyrotrophic hormone in blood
E. Estimation of rennin content in blood
• A 38 y.o. woman suffers from paroxysmal AP rises up to
240/120 mm Hg accompanied by nausea, vomiting,
tachycardia, excessive sweating. During the onset blood is
hyperglycemic. After the onset there is voluminous
urination. Kidneys sonography revealed accessory mass
bordering upon the upper pole of the right kidney;
presumably it belongs to the adrenal gland.
• What laboratory test will allow making a more precise
diagnosis?
A. Estimation of catecholamine and vanillyl-mandelic acid
excretion with urine
B. Estimation of insulin and C-peptide content in blood
C. Estimation of glomerular filtration rate
D. Estimation of thyroxin and thyrotrophic hormone in blood
E. Estimation of rennin content in blood
• A 49 y.o. female patient was admitted to the
hospital with acute attacks of headache
accompanied by pulsation in temples, AP rised
up to 280/140 mm Hg. Pheochromocytoma is
suspected.
• What mechanism of hypertensive atack does this
patient have?
A. Increasing of aldosterone level in blood
B. Increasing of plasma renin activity
C. Increasing of vasopressin excretion
D. Increasing of thyroxine excretion
E. Increasing of catecholamines concentration
• A 49 y.o. female patient was admitted to the
hospital with acute attacks of headache
accompanied by pulsation in temples, AP rised
up to 280/140 mm Hg. Pheochromocytoma is
suspected.
• What mechanism of hypertensive atack does this
patient have?
A. Increasing of aldosterone level in blood
B. Increasing of plasma renin activity
C. Increasing of vasopressin excretion
D. Increasing of thyroxine excretion
E. Increasing of catecholamines concentration
• A patient with suspected pheochromocytoma has
normal blood pressure in the periods between the
atacks and a tendency to tachycardia. Urine test
revealed no pathology. It was decided to use a
provocative test with histamine.
• What medication should be prepared to provide
emergency care in case of a positive test result?
A. Pipolphen
B. Phentolamine
C. Nifedipine
D. Mesatonum
E. Prednisolone
• A patient with suspected pheochromocytoma has
normal blood pressure in the periods between
the atacks and a tendency to tachycardia. Urine
test revealed no pathology. It was decided to use
a provocative test with histamine.
• What medication should be prepared to provide
emergency care in case of a positive test result?
A. Pipolphen
B. Nifedipine
C. Phentolamine
D. Mesatonum
E. Prednisolone
• A patient has been provisionally diagnosed with
pheochromocytoma at the stage of intermission.
BP is within norm, there is a tendency towards
tachycardia. No urine pathologies. The decision
has been made to perform a provocative test
with histamine.
• What drug should be kept close at hand for
emergency aid in case of positive test result?
A. Phentolamine
B. PipolphenB
C. Nifedipine
D. Mesaton (Phenylephrine)
E. Prednisolone
• A patient has been provisionally diagnosed with
pheochromocytoma at the stage of intermission.
BP is within norm, there is a tendency towards
tachycardia. No urine pathologies. The decision
has been made to perform a provocative test
with histamine.
• What drug should be kept close at hand for
emergency aid in case of positive test result?
A. Phentolamine
B. PipolphenB
C. Nifedipine
D. Mesaton (Phenylephrine)
E. Prednisolone
• A 40 y.o. patient was diagnosed: 1. Medullar
thyroid gland cancer. 2. Pheochromocytoma.
• What operation should be performed at first?
A. Operation on thyroid gland
B. Operation on account of pheochromocytoma
C. Krail’s operation
D. Subtotal resection of thyroid gland and
fascicular resection of lymphatic nodes
E. Vanach’s operation
• A 40 y.o. patient was diagnosed: 1. Medullar
thyroid gland cancer. 2. Pheochromocytoma.
• What operation should be performed at first?
A. Operation on thyroid gland
B. Operation on account of
pheochromocytoma
C. Krail’s operation
D. Subtotal resection of thyroid gland and
fascicular resection of lymphatic nodes
E. Vanach’s operation
Step
• 45. A 42-year-old man comes to the office for preoperative evaluation prior to
undergoing adrenalectomy scheduled in 2 weeks. One month ago, he received
care in the emergency department for pain over his right flank following a motor
vehicle collision. At that time, blood pressure was 160/100 mm Hg and CT scan of
the abdomen showed an incidental 10-cm left adrenal mass. Results of laboratory
studies, including complete blood count, serum electrolyte concentrations, and
liver function tests, were within the reference ranges. The patient otherwise had
been healthy and had never been told that he had elevated blood pressure. He
takes no medications. A follow-up visit in the office 2 weeks ago disclosed elevated
urinary normetanephrine and metanephrine and plasma aldosterone
concentrations. The patient was referred to a surgeon, who recommended the
adrenalectomy. Today, vital signs are temperature 36.6°C (97.9°F), pulse 100/min,
respirations 14/min, and blood pressure 170/95 mm Hg. Physical examination
discloses no significant findings. Initial preoperative preparation should include
treatment with which of the following?
A. Labetalol
B. A loading dose of potassium chloride
C. Nifedipine
D. Phenoxybenzamine
E. Spironolactone
• 45. A 42-year-old man comes to the office for preoperative evaluation
prior to undergoing adrenalectomy scheduled in 2 weeks. One month ago,
he received care in the emergency department for pain over his right flank
following a motor vehicle collision. At that time, blood pressure was
160/100 mm Hg and CT scan of the abdomen showed an incidental 10-cm
left adrenal mass. Results of laboratory studies, including complete blood
count, serum electrolyte concentrations, and liver function tests, were
within the reference ranges. The patient otherwise had been healthy and
had never been told that he had elevated blood pressure. He takes no
medications. A follow-up visit in the office 2 weeks ago disclosed elevated
urinary normetanephrine and metanephrine and plasma aldosterone
concentrations. The patient was referred to a surgeon, who recommended
the adrenalectomy. Today, vital signs are temperature 36.6°C (97.9°F),
pulse 100/min, respirations 14/min, and blood pressure 170/95 mm Hg.
Physical examination discloses no significant findings. Initial preoperative
preparation should include treatment with which of the following?
A. Labetalol
B. A loading dose of potassium chloride
C. Nifedipine
D. Phenoxybenzamine
E. Spironolactone
• 23. A 30-year-old woman comes to the physician because of
intermittent throbbing headaches, sweating, and pallor over the
past 3 months. She has had several blood pressure measurements
that fluctuate from 110/80 mm Hg to 160/108 mm Hg. Her pulse is
100/min, and blood pressure now is 138/88 mm Hg.
• Serum studies show: Na+ 140 mEq/L Cl− 110 mEq/L K + 4.5 mEq/L
HCO3 − 26 mEq/L Urea nitrogen 14 mg/dL Creatinine 1 mg/dL
• Which of the following is the most likely location of the
abnormality?
A. Adrenal cortex
B. Adrenal medulla
C. Aorta
D. Renal arterioles
E. Renal glomeruli
F. Thyroid gland
• 23. A 30-year-old woman comes to the physician because of
intermittent throbbing headaches, sweating, and pallor over the
past 3 months. She has had several blood pressure measurements
that fluctuate from 110/80 mm Hg to 160/108 mm Hg. Her pulse is
100/min, and blood pressure now is 138/88 mm Hg.
• Serum studies show: Na+ 140 mEq/L Cl− 110 mEq/L K + 4.5 mEq/L
HCO3 − 26 mEq/L Urea nitrogen 14 mg/dL Creatinine 1 mg/dL
• Which of the following is the most likely location of the
abnormality?
A. Adrenal cortex
B. Adrenal medulla
C. Aorta
D. Renal arterioles
E. Renal glomeruli
F. Thyroid gland
Thyroid storm
Predisposing factors  Diagnostic criteria
 seen shortly after thyroidectomy
or 131I therapy in an inadequately
prepared patient;
 severe infection in patients with
previously unrecognized or
inadequately treated
hyperthyroidism;
 Trauma; surgery;
 embolism, cerebrovascular
accidents or acute cirinary
syndrome;
 diabetic acidosis;
 toxemia of pregnancy; labor;
 withdrawal of antithyroid
medication;
 radiation thyroiditis.
• fever;
• dehydration;
• extreme restlessness with wide emotional
swings;
• confusion;
• delirium, psychosis or even coma;
• hepatomegaly with mild jaundice;
• tachycardia, atrial fibrillation
• marked weakness and muscle wasting;
cardiac failure,
• the patient may present with
cardiovascular collapse or shock
• On the first day after a surgery for diffuse toxic
goiter a patient developed difficulty breathing,
cold sweats, weakness. Objectively: pale skin,
body temperature - 38, 5oC , RR - 25/min, Ps-
110/min, AP-90/60 mm Hg.
• What early postoperative complication occurred
in the patient?
A. Thyrotoxic crisis
B. Hypothyroid crisis
C. Postoperative tetany
D. Acute thyroiditis
E. Compression of the trachea by the hematoma
• On the first day after a surgery for diffuse toxic
goiter a patient developed di-fficulty breathing,
cold sweats, weakness. Objectively: pale skin,
body temperature - 38, 5oC , RR - 25/min, Ps-
110/min, AP-90/60 mm Hg. What early
postoperative complication occurred in the
patient?
A. Thyrotoxic crisis
B. Hypothyroid crisis
C. Postoperative tetany
D. Acute thyroiditis
E. Compression of the trachea by the hematoma
Myxedema coma
Predisposing factors  Diagnostic criteria
• exposure to cold;
• acute infection;
• trauma;
• myocardial infarction, cardiac failure;
• drugs (sedatives, barbiturates,
amiodarone, anesthetic agents,
diuretics, beta-blockers);
• hyponatremia, hypoxia, hypercapnea.
• extreme hypothermia (temperatures
24 to 320C), areflexia, seizures, CO2
retention, and respiratory depression
caused by decreased cerebral blood
flow. Severe hypothermia may be
missed unless special low reading
thermometers are used. Rapid
diagnosis (based on clinical judgement,
history, and physical examination) is
imperative because early death is
likely.
• Neuropsychiatric manifestation:
develops psychosis with delusions and
hallucinations (“myxedema madness”),
progressing to depressed level of
consciousness, convulsions and coma.
• A 39-year-old female patient complains of rapid
fatigability, drowsiness, dry skin, hair loss,
swelling of the face. A month ago, she
underwent a surgery for thyrotoxicosis.
• The patient has the following gland dysfunction:
A. Thyroid (hypothyroidism), due to inadequate
operative technique
B. Pituitary, due to a tumor
C. Adrenal
D. Parathyroid, due to the gland removal during
surgery
E. Ovarian, due to a tumor
• A 39-year-old female patient complains of rapid
fatigability, drowsiness, dry skin, hair loss,
swelling of the face. A month ago, she underwent
a surgery for thyrotoxicosis.
• The patient has the following gland dysfunction:
A. Thyroid (hypothyroidism), due to inadequate
operative technique
B. Pituitary, due to a tumor
C. Adrenal
D. Parathyroid, due to the gland removal during
surgery
E. Ovarian, due to a tumor
Step
• 62. A 62-year-old woman comes to the physician for a routine
health maintenance examination. On questioning, she has had
fatigue, constipation, and a 9-kg (20-lb) weight gain during the past
year. She receives estrogen replacement therapy. Serum lipid
studies were within the reference range 5 years ago. She is 157 cm
(5 ft 2 in) tall and weighs 77 kg (170 lb); BMI is 31 kg/m2 . Physical
examination shows no other abnormalities.
• Serum lipid studies today show: Total cholesterol 269 mg/dL HDL-
cholesterol 48 mg/dL LDL-cholesterol 185 mg/dL Triglycerides 180
mg/dL
• Which of the following is the most likely cause?
A. Alcohol
B. Diabetes mellitus
C. Estrogen deficiency
D. Estrogen replacement therapy
E. Hypothyroidism
F. Thiazide diuretic therapy
• 62. A 62-year-old woman comes to the physician for a routine health
maintenance examination. On questioning, she has had fatigue,
constipation, and a 9-kg (20-lb) weight gain during the past year. She
receives estrogen replacement therapy. Serum lipid studies were within
the reference range 5 years ago. She is 157 cm (5 ft 2 in) tall and weighs 77
kg (170 lb); BMI is 31 kg/m2 . Physical examination shows no other
abnormalities.
• Serum lipid studies today show: Total cholesterol 269 mg/dL HDL-
cholesterol 48 mg/dL LDL-cholesterol 185 mg/dL Triglycerides 180 mg/dL
• Which of the following is the most likely cause?
A. Alcohol
B. Diabetes mellitus
C. Estrogen deficiency
D. Estrogen replacement therapy
E. Hypothyroidism
F. Thiazide diuretic therapy
Hypoparathyroid crise
Etiology Clinical signs Laboratory findings
• Postoperative: due to
damage to the thyroid
gland during thyroid
surgery (commonest
cause);
• Idiopathic
• Pseudohypoparathyroidi
sm (resistance to PTH)
• Acute severe,
hypocalcemia may
cause laryngospasm,
confusion, seizures,
vascular collapse with
bradycardia and
decompensated heart
failure.
• Acute, moderate
hypocalcemia may
cause increased
excitability of nerves
and muscules, leading
to circumoral or distal
parasthesia and tetany.
• Positive Chvostek’s and
Trousseau’s signs
• serum calcium is low
• serumPTH is low (or
inappropriatery normal
in the setting of
hypocalcemia is
indicative of
hypoparathyroidism: a
high levelof PTH is often
found in patients with
vitamin D deficiency,
PTH resistance, and
hyperphosphatemia);
• serum phosphate is
high
• A 63 y.o. patient was operated on account of big
multinodular euthyroid goiter. Despite of techical
difficulties a forced subtotal resection of both parts of
the thyroid gland was performed. On the 4-th day
after the operation the woman had cramps of face
muscles and upper extremities, stomach ache.
Positive Chvostek’s and Trousseau’s signs.
• What is the most probable cause of such condition?
A. Insufficiency of parathyroid glands
B. Postoperative hypothyroidism
C. Thyrotoxic crisis
D. Injury of recurrent nerve
E. Tracheomalacia
• A 63 y.o. patient was operated on account of big
multinodular euthyroid goiter. Despite of techical
difficulties a forced subtotal resection of both parts of
the thyroid gland was performed. On the 4-th day after
the operation the woman had cramps of face muscles
and upper extremities, stomach ache. Positive
Chvostek’s and Trousseau’s signs.
• What is the most probable cause of such condition?
A. Insufficiency of parathyroid glands
B. Postoperative hypothyroidism
C. Thyrotoxic crisis
D. Injury of recurrent nerve
E. Tracheomalacia
Step
• 36. 60-year-old man had a total thyroidectomy and excision of enlarged
left jugular lymph nodes for follicular carcinoma. The operation was
uncomplicated. He is receiving intravenous 5% dextrose and 0.45%
saline with potassium. Twelve hours after the operation he develops
circumoral numbness and paresthesias in his fingertips, and he
becomes very anxious. Vital signs are temperature 37.6°C (99.7°F),
pulse 90/min, respirations 16/min, and blood pressure 140/90 mm Hg.
Physical examination discloses a dry neck dressing and no stridor.
Extremities are warm, with brisk capillary refill time.
• Additional physical examination is most likely to show which of the
following?
A. Babinski sign present bilaterally
B. Chvostek sign
C. Deviation of the tongue to the left side
D. A drooping left shoulder
E. Hyporeflexia
• 36. 60-year-old man had a total thyroidectomy and excision
of enlarged left jugular lymph nodes for follicular
carcinoma. The operation was uncomplicated. He is
receiving intravenous 5% dextrose and 0.45% saline with
potassium. Twelve hours after the operation he develops
circumoral numbness and paresthesias in his fingertips, and
he becomes very anxious. Vital signs are temperature
37.6°C (99.7°F), pulse 90/min, respirations 16/min, and
blood pressure 140/90 mm Hg. Physical examination
discloses a dry neck dressing and no stridor. Extremities are
warm, with brisk capillary refill time.
• Additional physical examination is most likely to show
which of the following?
A. Babinski sign present bilaterally
B. Chvostek sign
C. Deviation of the tongue to the left side
D. A drooping left shoulder
E. Hyporeflexia
Anurea
Oliguria - Urine output in the amount of <500 ml per day
Anuria - Urine output in the amount of <50 ml per day
The main reasons for AKI Absolute indications towards for
renal replacement therapy in AKD
1. Inadequate perfusion of
kidneys (shock, collapse,
acute bleeding, renal artery
thrombosis, etc.).
2. Inflammatory and
nephrotoxic factors
(infections, poisoning with
mushrooms, heavy metals
salts, chloric hydrocarbons,
etc.).
3. Nephrotoxic drugs (drugs, X-
ray contrast agents).
4. Obstruction (occlusion) of
urinary tracts (tumors,
urolithiasis, strictures,
ligatures).
• Anuria(≤100 ml/24 hr or ≤50 ml/12 hr).
• Decompensated metabolic acidosis (рН
<7.1 mmol/L).
• Azotemia (blood urea >30 mmol/L or blood
creatinine >350 µmol/L).
• Dysnatriemia (115> Na+ >160 mmol/L).
• Hyperpotassemia (К+>6.5 mmol/L).
• Hypermagnesemia 4 mmol/L and more in
patients with anuria and absence of deep
tendon reflexes.
• Hyperhydratation with diuretic resistance
or the risk of pulmonary edema and/or the
brain edema.
• Overdose of dialysate drugs.
• Coagulopathy, which requires infusion of a
large number of blood products in a patient
with a risk of pulmonary edema or acute
respiratory distress syndrome.
• Lactate acidosis as a result of metformin
• 2 weeks after recovering from tonsillitis an 8-year-old boy
developed edemas of face and lower limbs. Objectively: the
patient is in grave condition, BP - 120/80 mm Hg. Urine is
of dark brown colour. Oliguria is present. On urine analysis:
specific gravity - 1,015, protein - 1,2 g/l, RBCs are leached
and cover the whole vision field, granular casts - 1-2 in the
vision field, salts are represented by urates (large quantity).
• What is the most likely diagnosis?
A. Acute glomerulonephritis with nephritic syndrome
B. Acute glomerulonephritis with nephrotic syndrome
C. Acute glomerulonephritis with nephrotic syndrome,
hematuria and hypertension
D. Acute glomerulonephritis with isolated urinary syndrome
E. Nephrolithiasis
• 2 weeks after recovering from tonsillitis an 8-year-old boy
developed edemas of face and lower limbs. Objectively: the
patient is in grave condition, BP - 120/80 mm Hg. Urine is of
dark brown colour. Oliguria is present. On urine analysis:
specific gravity - 1,015, protein - 1,2 g/l, RBCs are leached
and cover the whole vision field, granular casts - 1-2 in the
vision field, salts are represented by urates (large quantity).
• What is the most likely diagnosis?
A. Acute glomerulonephritis with nephritic syndrome
B. Acute glomerulonephritis with nephrotic syndrome
C. Acute glomerulonephritis with nephrotic syndrome,
hematuria and hypertension
D. Acute glomerulonephritis with isolated urinary syndrome
E. Nephrolithiasis
• A 50 y.o. woman who suffers from chronic
pyelonephritis was prescribed a combination of
antibiotics for the period of exacerbation - gentamicin
(80 mg 3 times a day) and biseptol (960 mg twice a
day).
• What consequences may be caused by such a
combination of antibiotics?
A. Glomerulosclerosis
B. Acute renal insufficiency
C. Chronic renal insufficiency
D. Antibiotic combination is optimal and absolutely safe
E. Acute suprarenal insufficiency
• A 50 y.o. woman who suffers from chronic
pyelonephritis was prescribed a combination of
antibiotics for the period of exacerbation - gentamicin
(80 mg 3 times a day) and biseptol (960 mg twice a
day).
• What consequences may be caused by such a
combination of antibiotics?
A. Glomerulosclerosis
B. Acute renal insufficiency
C. Chronic renal insufficiency
D. Antibiotic combination is optimal and absolutely safe
E. Acute suprarenal insufficiency
• A 33 year old patient has acute blood loss
(erythrocytes - 2,2·1012/l, Hb- 55 g/l), blood group is
A(II)Rh+. Accidentally the patient go tо transfusion of
donor pack edred blood cells of AB(IV) Rh+group. An
hour later the patient became anxious, got abdominal
andlumbar pain. Ps- 134 bpm, AP- 100/65 mm Hg,
body temperature - 38,6oC. After catheterization of
urinary bladder 12 ml/h of dark-brown urine were
obtained.
• What complication is it?
A. Cardiac shock
B. Acute renal insufficiency
C. Allergic reaction to the donor red blood cells
D. Citrate intoxication
E. Toxic infectious shock
• A 33 year old patient has acute blood loss
(erythrocytes - 2,2·1012/l, Hb- 55 g/l), blood group is
A(II)Rh+. Accidentally the patient go tо transfusion of
donor pack edred blood cells of AB(IV) Rh+group. An
hour later the patient became anxious, got abdominal
andlumbar pain. Ps- 134 bpm, AP- 100/65 mm Hg,
body temperature - 38,6oC. After catheterization of
urinary bladder 12 ml/h of dark-brown urine were
obtained.
• What complication is it?
A. Cardiac shock
B. Acute renal insufficiency
C. Allergic reaction to the donor red blood cells
D. Citrate intoxication
E. Toxic infectious shock
• Among the following causes of acute renal
failure, the one that would be classified as
"postrenal" is:
A. Calculi
B. Cardiac failure
C. Septicemia
D. Rhabdomyolysis
E. Acute glomerulonephritis
• Among the following causes of acute renal
failure, the one that would be classified as
"postrenal" is:
A. Calculi
B. Cardiac failure
C. Septicemia
D. Rhabdomyolysis
E. Acute glomerulonephritis
• A 24 y.o. patient complains of nausea, vomiting, headache,
shortness of breath. He had an acute nephritis being 10
y.o. Proteinuria was found out in urine. Objectively: a skin is
grey-pale, the edema is not present. Accent of II tone
above aorta. BP 140/100-180/100 mm Hg. Blood level of
residual N2- 6,6 mmol/L, creatinine- 406 mmol/L. Day’s
diuresis-2300 ml, nocturia. Specific density of urine is 1009,
albumin- 0,9 g/L, WBC- 0-2 in f/vis. RBC.- single in f/vis.,
hyaline casts single in specimen.
• Your diagnosis?
A. Pheochromocytoma
B. Hypertensive illness of the II degree
C. Nephrotic syndrome
D. Chronic nephritis with violation of kidney function
E. Stenosis of kidney artery
• A 24 y.o. patient complains of nausea, vomiting, headache,
shortness of breath. He had an acute nephritis being 10 y.o.
Proteinuria was found out in urine. Objectively: a skin is
grey-pale, the edema is not present. Accent of II tone
above aorta. BP 140/100-180/100 mm Hg. Blood level of
residual N2- 6,6 mmol/L, creatinine- 406 mmol/L. Day’s
diuresis-2300 ml, nocturia. Specific density of urine is 1009,
albumin- 0,9 g/L, WBC- 0-2 in f/vis. RBC.- single in f/vis.,
hyaline casts single in specimen.
• Your diagnosis?
A. Pheochromocytoma
B. Hypertensive illness of the II degree
C. Nephrotic syndrome
D. Chronic nephritis with violation of kidney function
E. Stenosis of kidney artery
• A 39-year-old man complains of morning headaches,
appetite loss, nausea, morning vomiting, periodic nasal
hemorrhages. The patient had a case of acute
glomerulonephritis at the age of 15. Examination revealed
rise of arterial pressure up to 220/130 mm Hg, skin
hemorrhages on his arms and legs, pallor of skin and
mucous membranes.
• What biochemical parameter is the most important for
making diagnosis in this case?
A. Blood creatinine*
B. Blood bilirubin
C. Blood sodium
D. Uric acid
E. Fibrinoge
• A 39-year-old man complains of morning headaches,
appetite loss, nausea, morning vomiting, periodic nasal
hemorrhages. The patient had a case of acute
glomerulonephritis at the age of 15. Examination revealed
rise of arterial pressure up to 220/130 mm Hg, skin
hemorrhages on his arms and legs, pallor of skin and
mucous membranes.
• What biochemical parameter is the most important for
making diagnosis in this case?
A. Blood creatinine*
B. Blood bilirubin
C. Blood sodium
D. Uric acid
E. Fibrinoge
• A 52 y.o. patient fell from 3 m hight on the
ground with the right lumbar area. He complains
of pain here. There is microhematuria in the
urine. On urography kidney’s functioning is
satisfactory.
• What is the most probable diagnosis?
A. Subcapsular kidney’s rupture
B. Multiple kidney’s ruptures
C. Paranephral hematoma
D. Kidney’s contusion
E. Kidney’s abruption
• A 52 y.o. patient fell from 3 m hight on the
ground with the right lumbar area. He complains
of pain here. There is microhematuria in the urea.
On urography kidney’s functioning is satisfactory.
• What is the most probable diagnosis?
A. Subcapsular kidney’s rupture
B. Multiple kidney’s ruptures
C. Paranephral hematoma
D. Kidney’s contusion
E. Kidney’s abruption
• A 35 y.o. male patient suffers from chronic
glomerulohephritis and has been on hemodialysis for
the last 3 years. He has developed irregularity in the
heart activity, hypotension, progressive weakness,
dyspnea. On ECG: bradycardia, 1st degree
atrioventicular block, high sharpened T-waves. Before
he had severely disturbed the drinking and diet
regimen.
• What is the most likely cause of these changes?
A. Hyperkalemia
B. Hyperhydratation
C. Hypokaliemia
D. Hypernatremia
E. Hypocalcaemia
• A 35 y.o. male patient suffers from chronic
glomerulohephritis and has been on hemodialysis for
the last 3 years. He has developed irregularity in the
heart activity, hypotension, progressive weakness,
dyspnea. On ECG: bradycardia, 1st degree
atrioventicular block, high sharpened T-waves. Before
he had severely disturbed the drinking and diet
regimen.
• What is the most likely cause of these changes?
A. Hyperkalemia
B. Hyperhydratation
C. Hypokaliemia
D. Hypernatremia
E. Hypocalcaemia
• A 3-year-old girl is being treated at a resuscitation
unit with diagnosis ”acute kidney failure, oligoanuric
stage”. ECG: high T wave, extended QRS complex,
displacement of S-T interval downwards below the
isoline.
• What electrolyte imbalance is it?
A. Hyperkalemia*
B. Hypokalemia
C. Hypocalcemia
D. Hypercalcemia
E. Hyperphosphatemia
• A 3-year-old girl is being treated at a resuscitation
unit with diagnosis ”acute kidney failure, oligoanuric
stage”. ECG: high T wave, extended QRS complex,
displacement of S-T interval downwards below the
isoline.
• What electrolyte imbalance is it?
A. Hyperkalemia
B. Hypokalemia
C. Hypocalcemia
D. Hypercalcemia
E. Hyperphosphatemia
• A 41-year-old male patient was delivered to a hospital
unconscious. During the previous 7 days he had been
taking large doses of biseptolum for a cold. The night
before, he began complaining of dyspnea, especially when
lying down, swollen legs, 2-day urinary retention. In the
morning he had seizures and lost consciousness. Objctively:
noisy breathing at the rate of 30/min, edematous legs and
lumbar region, Ps- 50/min. Plasma creatinine is 0,586
mmol/l, plasma potassium - 7,2 mmol/l.
• What treatment is necessary for this patient?
A. Hemodialysis
B. Large doses of verospiron
C. Plasma volume expanders
D. Glucocorticosteroids
E. Heparin
• A 41-year-old male patient was delivered to a hospital
unconscious. During the previ-ous 7 days he had been
taking large doses of biseptolum for a cold. The night
before, he began complaining of dyspnea, especially when
lying down, swollen legs, 2-day urinary retention. In the
morning he had seizures and lost consciousness. Objctively:
noisy breathi-ng at the rate of 30/min, edematous legs and
lumbar region, Ps- 50/min. Plasma creatinine is 0,586
mmol/l, plasma potassium - 7,2 mmol/l.
• What treatment is necessary for this patient?
A. Hemodialysis
B. Large doses of verospiron
C. Plasma volume expanders
D. Glucocorticosteroids
E. Heparin
• A 50-year-old male in a grave condition has been
admitted to the intensive care unit. It is known from
life history that the patient works in agriculture, and 3
hours ago was engaged into insecticide treatment of
crops for control of colorado potato beetle. Condition
on admission: acrocyanosis, bronchorrhea, tachypnea,
AP- 100/60 mm Hg, Ps- 44/min.
• What method of efferent therapy would be most
appropriate at this stage?
A. Hemosorbtion
B. Hemodialysis
C. Plasmapheresis
D. Lymphosorption
E. Plasma dialysis
• A 50-year-old male in a grave condition has been
admitted to the intensive care unit. It is known from
life history that the pati-ent works in agriculture, and 3
hours ago was engaged into insecticide treatment of
crops for control of colorado potato beetle. Condition
on admission: acrocyanosis, bronchorrhea, tachypnea,
AP- 100/60 mm Hg, Ps- 44/min.
• What method of efferent therapy would be most
appropriate at this stage?
A. Hemosorbtion
B. Hemodialysis
C. Plasmapheresis
D. Lymphosorption
E. Plasma dialysis
• A resuscitation unit received a 46-year-old woman, who has
been suffering from diabetes mellitus type 1 for
approximately 30 years. Objectively: the skin is pale, heart
sounds are weakened, BP is 170/100 mm Hg, lower limbs
are markedly swollen. Blood creatinine -1125 mcmol/l,
urea - 49,6 mmol/l, potassium - 6.3 mmol/l, glucose - 7,6
mmol/l, glomerular filtration rate - 5 ml/min.
• What treatment is indicated for the patient in the first
place?
A. Hemodialysis
B. Kidney transplantation
C. Hemofiltration
D. Enterosorption
E. Conservative detoxification therapy
• A resuscitation unit received a 46-year-old woman, who has
been suffering from diabetes mellitus type 1 for
approximately 30 years. Objectively: the skin is pale, heart
sounds are weakened, BP is 170/100 mm Hg, lower limbs
are markedly swollen. Blood creatinine -1125 mcmol/l,
urea - 49,6 mmol/l, potassium - 6.3 mmol/l, glucose - 7,6
mmol/l, glomerular filtration rate - 5 ml/min.
• What treatment is indicated for the patient in the first
place?
A. Hemodialysis
B. Kidney transplantation
C. Hemofiltration
D. Enterosorption
E. Conservative detoxification therapy
• A 30-year-old woman with a long history of chronic
pyelonephritis complains of considerable weakness,
sleepiness, decrease in diuresis down to 100 ml per
day. BP is 200/120 mm Hg. In blood: creatinine - 0,62
millimole/l, hypoproteinemia, albumines - 32 g/l,
potassium - 6,8 millimole/l, hypochromic anemia,
increased ESR.
• What is the first step in the patient treatment tactics?
A. Haemodialysis
B. Antibacterial therapy
C. Enterosorption
D. Haemosorption
E. Blood transfusion
• A 30-year-old woman with a long history of chronic
pyelonephritis complains of consi-derable weakness,
sleepiness, decrease in diuresis down to 100 ml per
day. BP is 200/120 mm Hg. In blood: creatinine - 0,62
millimole/l, hypoproteinemia, albumines - 32 g/l,
potassium - 6,8 millimole/l, hypochromic anemia,
increased ESR.
• What is the first step in the patient treatment tactics?
A. Haemodialysis
B. Antibacterial therapy
C. Enterosorption
D. Haemosorption
E. Blood transfusion
• A 23-year-old patient after intake of brake fluid
has developed anuria that has been lasting for 5
days already. Creatinine level increased up to
0,769 mmol/l.
• What treatment tactics should be chosen in the
given case?
A. Hemodialysis
B. Detoxification therapy
C. Antidotal therapy
D. Diuretics
E. Plasmapheresis
• A 23-year-old patient after intake of brake fluid
has developed anuria that has been lasting for 5
days already. Creatinine level increased up to
0,769 mmol/l.
• What treatment tactics should be chosen in the
given case?
A. Hemodialysis
B. Detoxification therapy
C. Antidotal therapy
D. Diuretics
E. Plasmapheresis
• A 35-year-old patient has been in the intensive care
unit for acute renal failure due to crush for 4 days.
Objectively: the patient is inadequate. Breathing rate -
32/min. Over the last 3 hours individual moist rales can
be auscultated in lungs. ECG shows high T waves, right
ventricular extrasystoles. CVP - 159 mm Hg. In blood:
the residual nitrogen - 62 mi-llimole/l, K+- 7,1
millimole/l, C l−- 78 mi-llimole/l, N a+- 120 millimole/l,
Ht - 0,32, Hb - 100 g/l, blood creatinine - 0,9 milli-
mole/l.
• The most appropriate method of treatment would be:
A. Hemodialysis
B. Plasma sorption
C. Hemosorption
D. Plasma filtration
E. Ultrafiltration
• A 35-year-old patient has been in the intensive care unit for
acute renal failure due to crush for 4 days. Objectively: the
patient is inadequate. Breathing rate - 32/min. Over the last
3 hours individual moist rales can be auscultated in lungs.
ECG shows high T waves, right ventri-cular extrasystoles.
CVP - 159 mm Hg. In blood: the residual nitrogen - 62 mi-
llimole/l, K+- 7,1 millimole/l, C l−- 78 mi-llimole/l, N a+- 120
millimole/l, Ht - 0,32, Hb - 100 g/l, blood creatinine - 0,9
milli-mole/l.
• The most appropriate method of treatment would be:
A. Hemodialysis
B. Plasma sorption
C. Hemosorption
D. Plasma filtration
E. Ultrafiltration
• A 30-year-old woman suffers from polycystic renal disease.
She has been admitted with signs of fatigue, thirst and
nocturia. Diuresis is up to 1800 ml per day. BP is 200/100
mm Hg. Blood test: erythrocytes -1,8 • 109/l, Hb- 68 g/l.
Urine analysis: specific gravity - 1005, leukocytes 50-60,
erythrocytes - 3-5 in the vision field, creatinine - 0,82
mmol/l, potassium - 6,5 mmol/l, glomerular filtration rate
- 10 ml/min.
• What tactics would be leading in the patient’s treatment?
A. Hemodialysis
B. Antibacterial therapy
C. Sorbent agents
D. Blood transfusion
E. Hypotensive therapy
• A 30-year-old woman suffers from polycystic renal disease.
She has been admitted with signs of fatigue, thirst and
nocturia. Diuresis is up to 1800 ml per day. BP is 200/100
mm Hg. Blood test: erythrocytes -1,8 • 109/l, Hb- 68 g/l.
Urine analysis: specific gravity - 1005, leukocytes 50-60,
erythrocytes - 3-5 in the vision field, creatinine - 0,82
mmol/l, potassium - 6,5 mmol/l, glomerular filtration rate
- 10 ml/min.
• What tactics would be leading in the patient’s treatment?
A. Hemodialysis
B. Antibacterial therapy
C. Sorbent agents
D. Blood transfusion
E. Hypotensive therapy
• A woman undergoing in-patient treatment for
viral hepatitis type B developed headache,
nausea, recurrent vomiting, memory lapses,
flapping tremor of her hands, rapid pulse. Sweet
smell from the mouth is detected. Body
temperature is 37,6°C, heart rate is 89/min.
• What complication developed in the patient?
A. Acute liver failure
B. Ischemic stroke
C. Gastrointestinal hemorrhage
D. Hypoglycemic shock
E. Meningoencephalitis
• A woman undergoing in-patient treatment for
viral hepatitis type B developed headache,
nausea, recurrent vomiting, memory lapses,
flapping tremor of her hands, rapid pulse. Sweet
smell from the mouth is detected. Body
temperature is 37,6°C, heart rate is 89/min.
• What complication developed in the patient?
A. Acute liver failure
B. Ischemic stroke
C. Gastrointestinal hemorrhage
D. Hypoglycemic shock
E. Meningoencephalitis
Thank You for attention!

More Related Content

What's hot

Management of hyperprolactinemic disorders
Management of hyperprolactinemic disordersManagement of hyperprolactinemic disorders
Management of hyperprolactinemic disordersMohamed Walaa El Deeb
 
Wilson’s disease
Wilson’s disease Wilson’s disease
Wilson’s disease PS Deb
 
Recent Trends in Gerd Management
Recent Trends in Gerd ManagementRecent Trends in Gerd Management
Recent Trends in Gerd ManagementHossam Ghoneim
 
Organic Acidemias Didactic Bb Day1 3 Jh
Organic Acidemias Didactic Bb Day1 3 JhOrganic Acidemias Didactic Bb Day1 3 Jh
Organic Acidemias Didactic Bb Day1 3 JhSIMD
 
Congenital Endocrinopathy
Congenital EndocrinopathyCongenital Endocrinopathy
Congenital Endocrinopathyankur meher
 
Liver diseases in pregnancy
Liver diseases in pregnancyLiver diseases in pregnancy
Liver diseases in pregnancyMohamed Albesh
 
Budd chiari syndrome
Budd chiari syndromeBudd chiari syndrome
Budd chiari syndromeJino Justin
 
ULCERATIVE COLITIS ( SEVERE) MANAGEMENT
ULCERATIVE COLITIS ( SEVERE) MANAGEMENT ULCERATIVE COLITIS ( SEVERE) MANAGEMENT
ULCERATIVE COLITIS ( SEVERE) MANAGEMENT Bhavin Mandowara
 
Jaundice Pathology
Jaundice PathologyJaundice Pathology
Jaundice PathologyAnurag Yadav
 
Necrotizing enterocolitis
Necrotizing enterocolitisNecrotizing enterocolitis
Necrotizing enterocolitisAfnan Shamraiz
 
Esophageal atresia & tracheo-esophageal fistula
Esophageal atresia & tracheo-esophageal fistulaEsophageal atresia & tracheo-esophageal fistula
Esophageal atresia & tracheo-esophageal fistulazanzibul tareq
 
Achalasia Case presentation
Achalasia Case presentationAchalasia Case presentation
Achalasia Case presentationSandra saju
 

What's hot (20)

Management of hyperprolactinemic disorders
Management of hyperprolactinemic disordersManagement of hyperprolactinemic disorders
Management of hyperprolactinemic disorders
 
Wilson’s disease
Wilson’s disease Wilson’s disease
Wilson’s disease
 
Recent Trends in Gerd Management
Recent Trends in Gerd ManagementRecent Trends in Gerd Management
Recent Trends in Gerd Management
 
Approach to lft
Approach to lftApproach to lft
Approach to lft
 
Organic Acidemias Didactic Bb Day1 3 Jh
Organic Acidemias Didactic Bb Day1 3 JhOrganic Acidemias Didactic Bb Day1 3 Jh
Organic Acidemias Didactic Bb Day1 3 Jh
 
Congenital Endocrinopathy
Congenital EndocrinopathyCongenital Endocrinopathy
Congenital Endocrinopathy
 
Prolactin hormone
Prolactin hormoneProlactin hormone
Prolactin hormone
 
Male Hypogonadism, LOH,
Male Hypogonadism, LOH, Male Hypogonadism, LOH,
Male Hypogonadism, LOH,
 
Liver diseases in pregnancy
Liver diseases in pregnancyLiver diseases in pregnancy
Liver diseases in pregnancy
 
Drugs in pregnancy
Drugs in pregnancyDrugs in pregnancy
Drugs in pregnancy
 
Budd chiari syndrome
Budd chiari syndromeBudd chiari syndrome
Budd chiari syndrome
 
ULCERATIVE COLITIS ( SEVERE) MANAGEMENT
ULCERATIVE COLITIS ( SEVERE) MANAGEMENT ULCERATIVE COLITIS ( SEVERE) MANAGEMENT
ULCERATIVE COLITIS ( SEVERE) MANAGEMENT
 
Jaundice Pathology
Jaundice PathologyJaundice Pathology
Jaundice Pathology
 
Dipsi guidelines
Dipsi guidelinesDipsi guidelines
Dipsi guidelines
 
Necrotizing enterocolitis
Necrotizing enterocolitisNecrotizing enterocolitis
Necrotizing enterocolitis
 
Esophageal atresia & tracheo-esophageal fistula
Esophageal atresia & tracheo-esophageal fistulaEsophageal atresia & tracheo-esophageal fistula
Esophageal atresia & tracheo-esophageal fistula
 
hyperprolactinemia
hyperprolactinemiahyperprolactinemia
hyperprolactinemia
 
Puberty
PubertyPuberty
Puberty
 
Hyperprolactinema for undergraduate
Hyperprolactinema for undergraduateHyperprolactinema for undergraduate
Hyperprolactinema for undergraduate
 
Achalasia Case presentation
Achalasia Case presentationAchalasia Case presentation
Achalasia Case presentation
 

Similar to Hypoglycemic and hyperglycemic coma causes, signs, and treatment

Practical class 04 Acute complications of DM.ppt
Practical class 04 Acute complications of DM.pptPractical class 04 Acute complications of DM.ppt
Practical class 04 Acute complications of DM.pptidris85sham
 
Diabetic Ketoacidosis
Diabetic Ketoacidosis Diabetic Ketoacidosis
Diabetic Ketoacidosis Kerryn Rohit
 
Diabetic ketoacidosis by dr. noman
Diabetic ketoacidosis by dr. nomanDiabetic ketoacidosis by dr. noman
Diabetic ketoacidosis by dr. nomanAbdullah Al Noman
 
Diabetic keto acidosis
Diabetic keto acidosisDiabetic keto acidosis
Diabetic keto acidosisKumar Abhinav
 
Acute complications of Diabetes Mellitus
Acute complications of Diabetes MellitusAcute complications of Diabetes Mellitus
Acute complications of Diabetes MellitusAIIMS, New Delhi, India
 
Lecture presentation amls_lesson07_endocrine
Lecture presentation amls_lesson07_endocrineLecture presentation amls_lesson07_endocrine
Lecture presentation amls_lesson07_endocrinends1977
 
Adrenal insufficiency.pptx
Adrenal insufficiency.pptxAdrenal insufficiency.pptx
Adrenal insufficiency.pptxDoha Rasheedy
 
Diabetic emergencies
Diabetic emergenciesDiabetic emergencies
Diabetic emergencieshibboonline
 
DIABETIC KETOACIDOSIS PRESENTATION BY ROOMA KHALID
DIABETIC KETOACIDOSIS  PRESENTATION BY ROOMA KHALIDDIABETIC KETOACIDOSIS  PRESENTATION BY ROOMA KHALID
DIABETIC KETOACIDOSIS PRESENTATION BY ROOMA KHALIDRooma Khalid
 
Endocrine Emergencies.pptx
Endocrine Emergencies.pptxEndocrine Emergencies.pptx
Endocrine Emergencies.pptxmunriz
 
Theophylline toxicity
Theophylline toxicityTheophylline toxicity
Theophylline toxicityFadel Omar
 
Final acute complications of diabetes mellitus
Final  acute complications of diabetes mellitusFinal  acute complications of diabetes mellitus
Final acute complications of diabetes mellitusSandeep Yadav
 
Multisystem inflammatory syndrome with covid 19 in pediatrics
Multisystem inflammatory syndrome with covid 19 in pediatricsMultisystem inflammatory syndrome with covid 19 in pediatrics
Multisystem inflammatory syndrome with covid 19 in pediatricsMounika Bhallam
 
Anesthesia considration for DIABETES MELLITUS
Anesthesia considration for DIABETES MELLITUSAnesthesia considration for DIABETES MELLITUS
Anesthesia considration for DIABETES MELLITUSibrahimelkathiri1
 
Acute complications of diabetes
Acute complications of diabetesAcute complications of diabetes
Acute complications of diabetesJeyadeepa Ramaraj
 

Similar to Hypoglycemic and hyperglycemic coma causes, signs, and treatment (20)

Practical class 04 Acute complications of DM.ppt
Practical class 04 Acute complications of DM.pptPractical class 04 Acute complications of DM.ppt
Practical class 04 Acute complications of DM.ppt
 
DKA in children
DKA in childrenDKA in children
DKA in children
 
Diabetic Ketoacidosis
Diabetic Ketoacidosis Diabetic Ketoacidosis
Diabetic Ketoacidosis
 
Diabetic ketoacidosis by dr. noman
Diabetic ketoacidosis by dr. nomanDiabetic ketoacidosis by dr. noman
Diabetic ketoacidosis by dr. noman
 
Diabetic keto acidosis
Diabetic keto acidosisDiabetic keto acidosis
Diabetic keto acidosis
 
Diabetic Ketoacidosis.pptx
Diabetic Ketoacidosis.pptxDiabetic Ketoacidosis.pptx
Diabetic Ketoacidosis.pptx
 
Dka
DkaDka
Dka
 
Acute complications of Diabetes Mellitus
Acute complications of Diabetes MellitusAcute complications of Diabetes Mellitus
Acute complications of Diabetes Mellitus
 
Lecture presentation amls_lesson07_endocrine
Lecture presentation amls_lesson07_endocrineLecture presentation amls_lesson07_endocrine
Lecture presentation amls_lesson07_endocrine
 
Diabetic Emergencies.pptx
Diabetic Emergencies.pptxDiabetic Emergencies.pptx
Diabetic Emergencies.pptx
 
Adrenal insufficiency.pptx
Adrenal insufficiency.pptxAdrenal insufficiency.pptx
Adrenal insufficiency.pptx
 
Diabetic emergencies
Diabetic emergenciesDiabetic emergencies
Diabetic emergencies
 
DIABETIC KETOACIDOSIS PRESENTATION BY ROOMA KHALID
DIABETIC KETOACIDOSIS  PRESENTATION BY ROOMA KHALIDDIABETIC KETOACIDOSIS  PRESENTATION BY ROOMA KHALID
DIABETIC KETOACIDOSIS PRESENTATION BY ROOMA KHALID
 
Endocrine Emergencies.pptx
Endocrine Emergencies.pptxEndocrine Emergencies.pptx
Endocrine Emergencies.pptx
 
Theophylline toxicity
Theophylline toxicityTheophylline toxicity
Theophylline toxicity
 
Final acute complications of diabetes mellitus
Final  acute complications of diabetes mellitusFinal  acute complications of diabetes mellitus
Final acute complications of diabetes mellitus
 
Endocrine emergencies
Endocrine emergenciesEndocrine emergencies
Endocrine emergencies
 
Multisystem inflammatory syndrome with covid 19 in pediatrics
Multisystem inflammatory syndrome with covid 19 in pediatricsMultisystem inflammatory syndrome with covid 19 in pediatrics
Multisystem inflammatory syndrome with covid 19 in pediatrics
 
Anesthesia considration for DIABETES MELLITUS
Anesthesia considration for DIABETES MELLITUSAnesthesia considration for DIABETES MELLITUS
Anesthesia considration for DIABETES MELLITUS
 
Acute complications of diabetes
Acute complications of diabetesAcute complications of diabetes
Acute complications of diabetes
 

More from Ugo161BB

Tohouri Grace IM-638 Analgesics in Ob-gyn.pptx
Tohouri Grace IM-638 Analgesics in Ob-gyn.pptxTohouri Grace IM-638 Analgesics in Ob-gyn.pptx
Tohouri Grace IM-638 Analgesics in Ob-gyn.pptxUgo161BB
 
04_Modern problems of pregnancy induced hypertension-2019.pptx
04_Modern problems of pregnancy induced hypertension-2019.pptx04_Modern problems of pregnancy induced hypertension-2019.pptx
04_Modern problems of pregnancy induced hypertension-2019.pptxUgo161BB
 
casereport heart report 1.pptx
casereport heart report 1.pptxcasereport heart report 1.pptx
casereport heart report 1.pptxUgo161BB
 
Disease of Uterine Cervix and Various Cytology Tests.pptx
Disease of Uterine Cervix and Various Cytology Tests.pptxDisease of Uterine Cervix and Various Cytology Tests.pptx
Disease of Uterine Cervix and Various Cytology Tests.pptxUgo161BB
 
Presentation (1) 2 1.pptx
Presentation (1) 2 1.pptxPresentation (1) 2 1.pptx
Presentation (1) 2 1.pptxUgo161BB
 
acupuncture-therapy-center.pptx
acupuncture-therapy-center.pptxacupuncture-therapy-center.pptx
acupuncture-therapy-center.pptxUgo161BB
 
acupuncture ppt.pptx
acupuncture ppt.pptxacupuncture ppt.pptx
acupuncture ppt.pptxUgo161BB
 
Acute-Pancreatitis copy 1.pptx
Acute-Pancreatitis copy 1.pptxAcute-Pancreatitis copy 1.pptx
Acute-Pancreatitis copy 1.pptxUgo161BB
 
Diagnostic criteria and treatment of bronchiolitis.pptx
Diagnostic criteria and treatment of bronchiolitis.pptxDiagnostic criteria and treatment of bronchiolitis.pptx
Diagnostic criteria and treatment of bronchiolitis.pptxUgo161BB
 
Presentation (12) 1.pptx
Presentation (12) 1.pptxPresentation (12) 1.pptx
Presentation (12) 1.pptxUgo161BB
 
SEPSIS-1.pptx
SEPSIS-1.pptxSEPSIS-1.pptx
SEPSIS-1.pptxUgo161BB
 
30 Conditioned reflex activity_50d76961c784233ac88e667da6a6fab4.ppt
30 Conditioned reflex activity_50d76961c784233ac88e667da6a6fab4.ppt30 Conditioned reflex activity_50d76961c784233ac88e667da6a6fab4.ppt
30 Conditioned reflex activity_50d76961c784233ac88e667da6a6fab4.pptUgo161BB
 
epigenomics.ppt
epigenomics.pptepigenomics.ppt
epigenomics.pptUgo161BB
 
sepsis-powerpoint-slide-presentation---the-guidelines_-implementation-for-the...
sepsis-powerpoint-slide-presentation---the-guidelines_-implementation-for-the...sepsis-powerpoint-slide-presentation---the-guidelines_-implementation-for-the...
sepsis-powerpoint-slide-presentation---the-guidelines_-implementation-for-the...Ugo161BB
 
treatment of mitochondrial diseases.pptx
treatment of mitochondrial diseases.pptxtreatment of mitochondrial diseases.pptx
treatment of mitochondrial diseases.pptxUgo161BB
 

More from Ugo161BB (15)

Tohouri Grace IM-638 Analgesics in Ob-gyn.pptx
Tohouri Grace IM-638 Analgesics in Ob-gyn.pptxTohouri Grace IM-638 Analgesics in Ob-gyn.pptx
Tohouri Grace IM-638 Analgesics in Ob-gyn.pptx
 
04_Modern problems of pregnancy induced hypertension-2019.pptx
04_Modern problems of pregnancy induced hypertension-2019.pptx04_Modern problems of pregnancy induced hypertension-2019.pptx
04_Modern problems of pregnancy induced hypertension-2019.pptx
 
casereport heart report 1.pptx
casereport heart report 1.pptxcasereport heart report 1.pptx
casereport heart report 1.pptx
 
Disease of Uterine Cervix and Various Cytology Tests.pptx
Disease of Uterine Cervix and Various Cytology Tests.pptxDisease of Uterine Cervix and Various Cytology Tests.pptx
Disease of Uterine Cervix and Various Cytology Tests.pptx
 
Presentation (1) 2 1.pptx
Presentation (1) 2 1.pptxPresentation (1) 2 1.pptx
Presentation (1) 2 1.pptx
 
acupuncture-therapy-center.pptx
acupuncture-therapy-center.pptxacupuncture-therapy-center.pptx
acupuncture-therapy-center.pptx
 
acupuncture ppt.pptx
acupuncture ppt.pptxacupuncture ppt.pptx
acupuncture ppt.pptx
 
Acute-Pancreatitis copy 1.pptx
Acute-Pancreatitis copy 1.pptxAcute-Pancreatitis copy 1.pptx
Acute-Pancreatitis copy 1.pptx
 
Diagnostic criteria and treatment of bronchiolitis.pptx
Diagnostic criteria and treatment of bronchiolitis.pptxDiagnostic criteria and treatment of bronchiolitis.pptx
Diagnostic criteria and treatment of bronchiolitis.pptx
 
Presentation (12) 1.pptx
Presentation (12) 1.pptxPresentation (12) 1.pptx
Presentation (12) 1.pptx
 
SEPSIS-1.pptx
SEPSIS-1.pptxSEPSIS-1.pptx
SEPSIS-1.pptx
 
30 Conditioned reflex activity_50d76961c784233ac88e667da6a6fab4.ppt
30 Conditioned reflex activity_50d76961c784233ac88e667da6a6fab4.ppt30 Conditioned reflex activity_50d76961c784233ac88e667da6a6fab4.ppt
30 Conditioned reflex activity_50d76961c784233ac88e667da6a6fab4.ppt
 
epigenomics.ppt
epigenomics.pptepigenomics.ppt
epigenomics.ppt
 
sepsis-powerpoint-slide-presentation---the-guidelines_-implementation-for-the...
sepsis-powerpoint-slide-presentation---the-guidelines_-implementation-for-the...sepsis-powerpoint-slide-presentation---the-guidelines_-implementation-for-the...
sepsis-powerpoint-slide-presentation---the-guidelines_-implementation-for-the...
 
treatment of mitochondrial diseases.pptx
treatment of mitochondrial diseases.pptxtreatment of mitochondrial diseases.pptx
treatment of mitochondrial diseases.pptx
 

Recently uploaded

Call Girl Bangalore Nandini 7001305949 Independent Escort Service Bangalore
Call Girl Bangalore Nandini 7001305949 Independent Escort Service BangaloreCall Girl Bangalore Nandini 7001305949 Independent Escort Service Bangalore
Call Girl Bangalore Nandini 7001305949 Independent Escort Service Bangalorenarwatsonia7
 
Call Girls Yelahanka Bangalore 📲 9907093804 💞 Full Night Enjoy
Call Girls Yelahanka Bangalore 📲 9907093804 💞 Full Night EnjoyCall Girls Yelahanka Bangalore 📲 9907093804 💞 Full Night Enjoy
Call Girls Yelahanka Bangalore 📲 9907093804 💞 Full Night Enjoynarwatsonia7
 
💎VVIP Kolkata Call Girls Parganas🩱7001035870🩱Independent Girl ( Ac Rooms Avai...
💎VVIP Kolkata Call Girls Parganas🩱7001035870🩱Independent Girl ( Ac Rooms Avai...💎VVIP Kolkata Call Girls Parganas🩱7001035870🩱Independent Girl ( Ac Rooms Avai...
💎VVIP Kolkata Call Girls Parganas🩱7001035870🩱Independent Girl ( Ac Rooms Avai...Taniya Sharma
 
College Call Girls Pune Mira 9907093804 Short 1500 Night 6000 Best call girls...
College Call Girls Pune Mira 9907093804 Short 1500 Night 6000 Best call girls...College Call Girls Pune Mira 9907093804 Short 1500 Night 6000 Best call girls...
College Call Girls Pune Mira 9907093804 Short 1500 Night 6000 Best call girls...Miss joya
 
Vip Call Girls Anna Salai Chennai 👉 8250192130 ❣️💯 Top Class Girls Available
Vip Call Girls Anna Salai Chennai 👉 8250192130 ❣️💯 Top Class Girls AvailableVip Call Girls Anna Salai Chennai 👉 8250192130 ❣️💯 Top Class Girls Available
Vip Call Girls Anna Salai Chennai 👉 8250192130 ❣️💯 Top Class Girls AvailableNehru place Escorts
 
CALL ON ➥9907093804 🔝 Call Girls Baramati ( Pune) Girls Service
CALL ON ➥9907093804 🔝 Call Girls Baramati ( Pune)  Girls ServiceCALL ON ➥9907093804 🔝 Call Girls Baramati ( Pune)  Girls Service
CALL ON ➥9907093804 🔝 Call Girls Baramati ( Pune) Girls ServiceMiss joya
 
Call Girl Coimbatore Prisha☎️ 8250192130 Independent Escort Service Coimbatore
Call Girl Coimbatore Prisha☎️  8250192130 Independent Escort Service CoimbatoreCall Girl Coimbatore Prisha☎️  8250192130 Independent Escort Service Coimbatore
Call Girl Coimbatore Prisha☎️ 8250192130 Independent Escort Service Coimbatorenarwatsonia7
 
Call Girls Service In Shyam Nagar Whatsapp 8445551418 Independent Escort Service
Call Girls Service In Shyam Nagar Whatsapp 8445551418 Independent Escort ServiceCall Girls Service In Shyam Nagar Whatsapp 8445551418 Independent Escort Service
Call Girls Service In Shyam Nagar Whatsapp 8445551418 Independent Escort Serviceparulsinha
 
Call Girl Number in Vashi Mumbai📲 9833363713 💞 Full Night Enjoy
Call Girl Number in Vashi Mumbai📲 9833363713 💞 Full Night EnjoyCall Girl Number in Vashi Mumbai📲 9833363713 💞 Full Night Enjoy
Call Girl Number in Vashi Mumbai📲 9833363713 💞 Full Night Enjoybabeytanya
 
♛VVIP Hyderabad Call Girls Chintalkunta🖕7001035870🖕Riya Kappor Top Call Girl ...
♛VVIP Hyderabad Call Girls Chintalkunta🖕7001035870🖕Riya Kappor Top Call Girl ...♛VVIP Hyderabad Call Girls Chintalkunta🖕7001035870🖕Riya Kappor Top Call Girl ...
♛VVIP Hyderabad Call Girls Chintalkunta🖕7001035870🖕Riya Kappor Top Call Girl ...astropune
 
Bangalore Call Girls Marathahalli 📞 9907093804 High Profile Service 100% Safe
Bangalore Call Girls Marathahalli 📞 9907093804 High Profile Service 100% SafeBangalore Call Girls Marathahalli 📞 9907093804 High Profile Service 100% Safe
Bangalore Call Girls Marathahalli 📞 9907093804 High Profile Service 100% Safenarwatsonia7
 
VIP Call Girls Indore Kirti 💚😋 9256729539 🚀 Indore Escorts
VIP Call Girls Indore Kirti 💚😋  9256729539 🚀 Indore EscortsVIP Call Girls Indore Kirti 💚😋  9256729539 🚀 Indore Escorts
VIP Call Girls Indore Kirti 💚😋 9256729539 🚀 Indore Escortsaditipandeya
 
VIP Call Girls Pune Vrinda 9907093804 Short 1500 Night 6000 Best call girls S...
VIP Call Girls Pune Vrinda 9907093804 Short 1500 Night 6000 Best call girls S...VIP Call Girls Pune Vrinda 9907093804 Short 1500 Night 6000 Best call girls S...
VIP Call Girls Pune Vrinda 9907093804 Short 1500 Night 6000 Best call girls S...Miss joya
 
Call Girls Colaba Mumbai ❤️ 9920874524 👈 Cash on Delivery
Call Girls Colaba Mumbai ❤️ 9920874524 👈 Cash on DeliveryCall Girls Colaba Mumbai ❤️ 9920874524 👈 Cash on Delivery
Call Girls Colaba Mumbai ❤️ 9920874524 👈 Cash on Deliverynehamumbai
 
Artifacts in Nuclear Medicine with Identifying and resolving artifacts.
Artifacts in Nuclear Medicine with Identifying and resolving artifacts.Artifacts in Nuclear Medicine with Identifying and resolving artifacts.
Artifacts in Nuclear Medicine with Identifying and resolving artifacts.MiadAlsulami
 
Kesar Bagh Call Girl Price 9548273370 , Lucknow Call Girls Service
Kesar Bagh Call Girl Price 9548273370 , Lucknow Call Girls ServiceKesar Bagh Call Girl Price 9548273370 , Lucknow Call Girls Service
Kesar Bagh Call Girl Price 9548273370 , Lucknow Call Girls Servicemakika9823
 
(Rocky) Jaipur Call Girl - 9521753030 Escorts Service 50% Off with Cash ON De...
(Rocky) Jaipur Call Girl - 9521753030 Escorts Service 50% Off with Cash ON De...(Rocky) Jaipur Call Girl - 9521753030 Escorts Service 50% Off with Cash ON De...
(Rocky) Jaipur Call Girl - 9521753030 Escorts Service 50% Off with Cash ON De...indiancallgirl4rent
 

Recently uploaded (20)

Call Girl Bangalore Nandini 7001305949 Independent Escort Service Bangalore
Call Girl Bangalore Nandini 7001305949 Independent Escort Service BangaloreCall Girl Bangalore Nandini 7001305949 Independent Escort Service Bangalore
Call Girl Bangalore Nandini 7001305949 Independent Escort Service Bangalore
 
Call Girls Yelahanka Bangalore 📲 9907093804 💞 Full Night Enjoy
Call Girls Yelahanka Bangalore 📲 9907093804 💞 Full Night EnjoyCall Girls Yelahanka Bangalore 📲 9907093804 💞 Full Night Enjoy
Call Girls Yelahanka Bangalore 📲 9907093804 💞 Full Night Enjoy
 
💎VVIP Kolkata Call Girls Parganas🩱7001035870🩱Independent Girl ( Ac Rooms Avai...
💎VVIP Kolkata Call Girls Parganas🩱7001035870🩱Independent Girl ( Ac Rooms Avai...💎VVIP Kolkata Call Girls Parganas🩱7001035870🩱Independent Girl ( Ac Rooms Avai...
💎VVIP Kolkata Call Girls Parganas🩱7001035870🩱Independent Girl ( Ac Rooms Avai...
 
College Call Girls Pune Mira 9907093804 Short 1500 Night 6000 Best call girls...
College Call Girls Pune Mira 9907093804 Short 1500 Night 6000 Best call girls...College Call Girls Pune Mira 9907093804 Short 1500 Night 6000 Best call girls...
College Call Girls Pune Mira 9907093804 Short 1500 Night 6000 Best call girls...
 
Vip Call Girls Anna Salai Chennai 👉 8250192130 ❣️💯 Top Class Girls Available
Vip Call Girls Anna Salai Chennai 👉 8250192130 ❣️💯 Top Class Girls AvailableVip Call Girls Anna Salai Chennai 👉 8250192130 ❣️💯 Top Class Girls Available
Vip Call Girls Anna Salai Chennai 👉 8250192130 ❣️💯 Top Class Girls Available
 
CALL ON ➥9907093804 🔝 Call Girls Baramati ( Pune) Girls Service
CALL ON ➥9907093804 🔝 Call Girls Baramati ( Pune)  Girls ServiceCALL ON ➥9907093804 🔝 Call Girls Baramati ( Pune)  Girls Service
CALL ON ➥9907093804 🔝 Call Girls Baramati ( Pune) Girls Service
 
Call Girl Coimbatore Prisha☎️ 8250192130 Independent Escort Service Coimbatore
Call Girl Coimbatore Prisha☎️  8250192130 Independent Escort Service CoimbatoreCall Girl Coimbatore Prisha☎️  8250192130 Independent Escort Service Coimbatore
Call Girl Coimbatore Prisha☎️ 8250192130 Independent Escort Service Coimbatore
 
Call Girls Service In Shyam Nagar Whatsapp 8445551418 Independent Escort Service
Call Girls Service In Shyam Nagar Whatsapp 8445551418 Independent Escort ServiceCall Girls Service In Shyam Nagar Whatsapp 8445551418 Independent Escort Service
Call Girls Service In Shyam Nagar Whatsapp 8445551418 Independent Escort Service
 
Call Girl Number in Vashi Mumbai📲 9833363713 💞 Full Night Enjoy
Call Girl Number in Vashi Mumbai📲 9833363713 💞 Full Night EnjoyCall Girl Number in Vashi Mumbai📲 9833363713 💞 Full Night Enjoy
Call Girl Number in Vashi Mumbai📲 9833363713 💞 Full Night Enjoy
 
♛VVIP Hyderabad Call Girls Chintalkunta🖕7001035870🖕Riya Kappor Top Call Girl ...
♛VVIP Hyderabad Call Girls Chintalkunta🖕7001035870🖕Riya Kappor Top Call Girl ...♛VVIP Hyderabad Call Girls Chintalkunta🖕7001035870🖕Riya Kappor Top Call Girl ...
♛VVIP Hyderabad Call Girls Chintalkunta🖕7001035870🖕Riya Kappor Top Call Girl ...
 
sauth delhi call girls in Bhajanpura 🔝 9953056974 🔝 escort Service
sauth delhi call girls in Bhajanpura 🔝 9953056974 🔝 escort Servicesauth delhi call girls in Bhajanpura 🔝 9953056974 🔝 escort Service
sauth delhi call girls in Bhajanpura 🔝 9953056974 🔝 escort Service
 
Bangalore Call Girls Marathahalli 📞 9907093804 High Profile Service 100% Safe
Bangalore Call Girls Marathahalli 📞 9907093804 High Profile Service 100% SafeBangalore Call Girls Marathahalli 📞 9907093804 High Profile Service 100% Safe
Bangalore Call Girls Marathahalli 📞 9907093804 High Profile Service 100% Safe
 
VIP Call Girls Indore Kirti 💚😋 9256729539 🚀 Indore Escorts
VIP Call Girls Indore Kirti 💚😋  9256729539 🚀 Indore EscortsVIP Call Girls Indore Kirti 💚😋  9256729539 🚀 Indore Escorts
VIP Call Girls Indore Kirti 💚😋 9256729539 🚀 Indore Escorts
 
VIP Call Girls Pune Vrinda 9907093804 Short 1500 Night 6000 Best call girls S...
VIP Call Girls Pune Vrinda 9907093804 Short 1500 Night 6000 Best call girls S...VIP Call Girls Pune Vrinda 9907093804 Short 1500 Night 6000 Best call girls S...
VIP Call Girls Pune Vrinda 9907093804 Short 1500 Night 6000 Best call girls S...
 
Call Girls Colaba Mumbai ❤️ 9920874524 👈 Cash on Delivery
Call Girls Colaba Mumbai ❤️ 9920874524 👈 Cash on DeliveryCall Girls Colaba Mumbai ❤️ 9920874524 👈 Cash on Delivery
Call Girls Colaba Mumbai ❤️ 9920874524 👈 Cash on Delivery
 
Artifacts in Nuclear Medicine with Identifying and resolving artifacts.
Artifacts in Nuclear Medicine with Identifying and resolving artifacts.Artifacts in Nuclear Medicine with Identifying and resolving artifacts.
Artifacts in Nuclear Medicine with Identifying and resolving artifacts.
 
Kesar Bagh Call Girl Price 9548273370 , Lucknow Call Girls Service
Kesar Bagh Call Girl Price 9548273370 , Lucknow Call Girls ServiceKesar Bagh Call Girl Price 9548273370 , Lucknow Call Girls Service
Kesar Bagh Call Girl Price 9548273370 , Lucknow Call Girls Service
 
Russian Call Girls in Delhi Tanvi ➡️ 9711199012 💋📞 Independent Escort Service...
Russian Call Girls in Delhi Tanvi ➡️ 9711199012 💋📞 Independent Escort Service...Russian Call Girls in Delhi Tanvi ➡️ 9711199012 💋📞 Independent Escort Service...
Russian Call Girls in Delhi Tanvi ➡️ 9711199012 💋📞 Independent Escort Service...
 
Escort Service Call Girls In Sarita Vihar,, 99530°56974 Delhi NCR
Escort Service Call Girls In Sarita Vihar,, 99530°56974 Delhi NCREscort Service Call Girls In Sarita Vihar,, 99530°56974 Delhi NCR
Escort Service Call Girls In Sarita Vihar,, 99530°56974 Delhi NCR
 
(Rocky) Jaipur Call Girl - 9521753030 Escorts Service 50% Off with Cash ON De...
(Rocky) Jaipur Call Girl - 9521753030 Escorts Service 50% Off with Cash ON De...(Rocky) Jaipur Call Girl - 9521753030 Escorts Service 50% Off with Cash ON De...
(Rocky) Jaipur Call Girl - 9521753030 Escorts Service 50% Off with Cash ON De...
 

Hypoglycemic and hyperglycemic coma causes, signs, and treatment

  • 2. Hypoglycemic coma DKA HO Etiology Access of insulin in organism 1) related to diabetes: missed, delayed or inadequate meal; unexpected or unusual exercise; alcohol; errors in oral hypoglycemic agents or insulin dose/schedule/administration; 2)related to people with/without diabetes: chronic kidney disease IV-V stage; liver failure; counter regulatory failure: = adrenal failure; = hypopituitarism; = malabsorbtion; = insulinoma. Lack of insulin 1)newly diagnosed diabetes (presenting manifestation); 2)inadequate administration of exogenous insulin; 3)increased requirements for insulin caused by the presence of an underlying stressful condition: an intercurrent infection ; a vascular disorder (myocardial infarction, stroke); an endocrine disorder ; trauma; pregnancy; surgery. Dehydration of different etiology
  • 3. Hypoglycemic coma DKA HO Onset Rapid Gradual Gradual Clinical signs 1) adrenergic symptoms : = sweating; = nervousness; = tremulousness; = faintness; = anxiety; = palpitation; = hunger; 2) cerebral nervous system manifestations: = confusion; = dizziness; = inappropriate behavior (which can be mistaken for inebriation); = visual disturbances; = headache; = seizures; = loss of fine motor skills; = loss of consciousness. - Polydipsia, polyuria and weakness are the most common presenting complaints. - Anorexia, nausea, vomiting, and abdominal pain may be present and mimic an abdominal emergency. - Ileus and gastric dilatation may occur and predispose to aspiration. -Polyuria, polydipsia, weight loss, weakness - progressive changes in state of consciousness from mental cloudiness to coma - underlying conditions (such as cerebrovascular accident and subdural hematoma) - Seizures occur in 5 % of patients and may be either focal or generalized
  • 4. Hypoglycemic coma DKA HO Physical examination - The skin is cold, moist. -Full pulse. - Normal or raised blood pressure. - Shallow or normal breathing. - Hyperreflexia can be elicited. - Hypoglycemic coma is commonly associated with abnormally low body temperature - Patient may be unconsciousness. - The skin is dry - Hypothermia is common in DKA. A fever should be taken as strong evidence of infection. - Tachycardia frequently is present - Blood pressure is usually normal unless profound dehydration is present. - Hyperpnoea or Kussmaul respiration - acetone may be detected on the breath (musty (fruity) odor to the breath). - Hyporeflexia . - Severe dehydration is invariably present. - Various neurologic deficits (such as coma, transient hemiparesis, hyperreflexia, and generalized areflexia) are commonly present. - Altered states of consciousness from lethargy to coma are observed. - Findings associated with coexisting medical problems (e.g., renal disease, cardiovascular disease) may be evident
  • 5. Hypoglycemic coma DKA HO Laboratory features Low level of blood glucose - hyperglycemia; - ketonemia; - metabolic acidosis (plasma pH and bicarbonates are decreased, - potassium disturbances - Extreme hyperglycemia (blood glucose levels from 30 mmoll/l and over are common. - A markedly elevated serum osmolality is present, usually in excess of 350 mOsm/l. (Normal = 290 mOsm/l). - Serum ketones are usually not detectable, and patients are not acidic. - Serum sodium may be high - Serum potassium levels may be high (secondary to the effects of hyperosmolality as it draws potassium from the cells), normal, or low (from marked urinary losses from the osmotic diuresis). But potassium deficiency exists.
  • 6. • A 23 y.o. woman who suffers from insulin- dependent diabetes was admitted to the acute care department with mental confusion, inadequate anxious behaviour, hyperhidrosis, excessive salivation, tachycardia. • What examination will be a primary task? A. Clinical blood analysis B. Plasma electrolytes test C. Gaseous composition of arterial blood D. Blood test for sugar E. Blood urea and creatinine test
  • 7. • A 23 y.o. woman who suffers from insulin- dependent diabetes was admitted to the acute care department with mental confusion, inadequate anxious behaviour, hyperhidrosis, excessive salivation, tachycardia. • What examination will be a primary task? A. Clinical blood analysis B. Plasma electrolytes test C. Gaseous composition of arterial blood D. Blood test for sugar E. Blood urea and creatinine test
  • 8. • An unconscious patient presents with moist skin, shallow breathing. There are signs of previous injection on the shoulders and hips. BP- 110/70 mm Hg. Tonus of skeletal muscles and reflexes are increased. Cramps of muscles of the extremities are seen. • What is the most likely disorder? A. Hyperglycemic coma B. Hyperosmolar coma C. Hypoglycemic coma D. Hyperlactacidotic coma E. Stroke
  • 9. • An unconscious patient presents with moist skin, shallow breathing. There are signs of previous injection on the shoulders and hips. BP- 110/70 mm Hg. Tonus of skeletal muscles and reflexes are increased. Cramps of muscles of the extremities are seen. • What is the most likely disorder? A. Hyperglycemic coma B. Hyperosmolar coma C. Hypoglycemic coma D. Hyperlactacidotic coma E. Stroke
  • 10. • A 9 year old boy had acute respiratory viral infection. After it there appeared polydipsia, polyuria, weakness, nausea. Examination revealed the following symptoms: mental confusion, dry skin, soft eyeballs, Kussmaul’s respiration, acetone smell from the mouth, muffled heart sounds, soft and painless abdomen. Blood sugar was 19 millimole/l. • What acute condition is it? A. Acute renal insufficiency B. Hyperosmolar coma C. Cerebral coma D. Hepatic coma E. Ketoacidotic coma
  • 11. • A 9 year old boy had acute respiratory viral infection. After it there appeared polydipsia, polyuria, weakness, nausea. Examination revealed the following symptoms: mental confusion, dry skin, soft eyeballs, Kussmaul’s respiration, acetone smell from the mouth, muffled heart sounds, soft and painless abdomen. Blood sugar was 19 millimole/l. • What acute condition is it? A. Acute renal insufficiency B. Hyperosmolar coma C. Cerebral coma D. Hepatic coma E. Ketoacidic coma
  • 12. • A 3-year-old child has been diagnosed with type I diabetes mellitus, hyperosmolar coma. The laboratory confirmed the diagnosis. • Which laboratory findings are characteristic for such condition? A. High hyperglycemia without ketonemia B. Hyperglycemia and ketonemia C. Hyperglycemia and glucosuria D. Hyperglycemia and ketonuria E. Hyperglycemia and high indicators of acid-base balance
  • 13. • A 3-year-old child has been diagnosed with type I diabetes mellitus, hyperosmolar coma. The laboratory confirmed the diagnosis. • Which laboratory findings are characteristic for such condition? A. High hyperglycemia without ketonemia B. Hyperglycemia and ketonemia C. Hyperglycemia and glucosuria D. Hyperglycemia and ketonuria E. Hyperglycemia and high indicators of acid-base balance
  • 14. • An 8-year-old child with a 3-year-long history of diabetes was hospitalized in hyperglycemic coma. • Specify the initial dose of insulin to be administered: A. 0,1-0,2 U/kg of body weight per hour B. A. 0,05 U/kg of body weight per hour C. B. 0,2-0,3 U/kg of body weight per hour D. 0,3-0,4 U/kg of body weight per hour E. 0,4-0,5 U/kg of body weight per hour
  • 15. • An 8-year-old child with a 3-year-long history of diabetes was hospitalized in hyperglycemic coma. • Specify the initial dose of insulin to be administered: A. 0,1-0,2 U/kg of body weight per hour B. A. 0,05 U/kg of body weight per hour C. B. 0,2-0,3 U/kg of body weight per hour D. 0,3-0,4 U/kg of body weight per hour E. 0,4-0,5 U/kg of body weight per hour
  • 16. • During examination a patient is unconscious, his skin is dry and hot, face hyperemia is present. The patient has Kussmaul’s respiration, there is also smell of acetone in the air. Symptoms of peritoneum irritation are positive. Blood sugar is 33 mmol/l. • What emergency actions should be taken? A. Intravenous infusion of short-acting insulin B. Intravenous infusion of glucose along with insulin C. Introduction of long-acting insulin D. Intravenous infusion of neohaemodesum along with glutamic acid E. Intravenous infusion of sodium chloride saline
  • 17. • During examination a patient is unconsci-ous, his skin is dry and hot, face hyperemia is present. The patient has Kussmaul’s respiration, there is also smell of acetone in the air. Symptoms of peritoneum irritation are positive. Blood sugar is 33 mmol/l. • What emergency actions should be taken? A. Intravenous infusion of short-acting insulin B. Intravenous infusion of glucose along with insulin C. Introduction of long-acting insulin D. Intravenous infusion of neohaemodesum along with glutamic acid E. Intravenous infusion of sodium chloride saline
  • 18. • A 23-year-old woman presents with diabetes mellitus type 1. She complains of weakness, headache, nausea, and vomiting. Objectively: temperature is 37,6°C, heart rate is 98/min., BP is 95/65 mm Hg, respiration rate is 32/min., loud. Smell of acetone is detected, heart sounds are muffled, pulse is rhythmic. The stomach is sensitive in the epigastrium. Costovertebral angle tenderness (Murphy’s punch sign) is present. Blood glucose is 28,5 mmol/l; blood leukocytes - 16, 5 • 109/l. In urine: acetone ++, leukocytes - 25-40 in the vision field. Blood pH is 7,1. • What correction method would be the most advisable? A. Infusion of normal saline + insulinotherapy in small portions* B. Infusion of 5% glucose + insulinotherapy in large portions C. Infusion of 2,5% sodium bicarbonate + insulinotherapy D. Intravenous administration of antibiotics + insulinotherapy E. Infusion of dextran solutions + insuli-notherapy
  • 19. • A 23-year-old woman presents with diabetes mellitus type 1. She complains of weakness, headache, nausea, and vomiting. Objectively: temperature is 37,6°C, heart rate is 98/min., BP is 95/65 mm Hg, respiration rate is 32/min., loud. Smell of acetone is detected, heart sounds are muffled, pulse is rhythmic. The stomach is sensitive in the epigastrium. Costovertebral angle tenderness (Murphy’s punch sign) is present. Blood glucose is 28,5 mmol/l; blood leukocytes - 16, 5 • 109/l. In urine: acetone ++, leukocytes - 25-40 in the vision field. Blood pH is 7,1. • What correction method would be the most advisable? A. Infusion of normal saline + insulinotherapy in small portions B. Infusion of 5% glucose + insulinotherapy in large portions C. Infusion of 2,5% sodium bicarbonate + insulinotherapy D. Intravenous administration of antibiotics + insulinotherapy E. Infusion of dextran solutions + insuli-notherapy
  • 20. • A 23-year-old woman after stress has developed thirst, polydipsia, polyuria, weight loss, increasing fatigue. Later she developed nausea and somnolence, lost consciousness and was hospitalised. Glycemia is 27 mmol/l, acetone in urine is sharply positive. Treatment for ketoacidotic coma was initiated. • When would it be advisable to start preventive treatment of hypoglycemia by introduction of 5% glucose solution? A. After glycemia rate drops to 13-14 mmol/l B. 2 hours after beginning of insulinotherapy C. When patient becomes conscious D. After glycemia rate becomes normal E. If glycemia decreases with the rate over 5 mmol/l per нour
  • 21. • A 23-year-old woman after stress has developed thirst, polydipsia, polyuria, weight loss, increasing fatigue. Later she developed nausea and somnolence, lost consciousness and was hospitalised. Glycemia is 27 mmol/l, acetone in urine is sharply positive. Treatment for ketoacidotic coma was initiated. • When would it be advisable to start preventive treatment of hypoglycemia by introduction of 5% glucose solution? A. After glycemia rate drops to 13-14 mmol/l B. 2 hours after beginning of insulinotherapy C. When patient becomes conscious D. After glycemia rate becomes normal E. If glycemia decreases with the rate over 5 mmol/l per hour
  • 22. • A 72-year-old woman suffers from diabetes mellitus type 2, concomitant diseases are stage 2 hypertension and stage 2B heart failure. She takes metformin. Hypertensic crisis had occurred the day before, after which the patient developed extreme weakness, myalgias, thirst, dry mouth, polyuria. BP is 140/95 mm Hg, heart rate is 98/min., no edemas or smell of acetone detected. • What measures should be taken to prevent development of comatose state in the patient? A. Stop metformin, prescribe short-acting insulin B. Double the dosage of metformin C. Apply hypotonic solution of sodium chloride D. Additionally prescribe long-acting insulin E. Prescribe glibenclamid
  • 23. • A 72-year-old woman suffers from diabetes mellitus type 2, concomitant diseases are stage 2 hypertension and stage 2B heart failure. She takes metformin. Hypertensic crisis had occurred the day before, after which the patient developed extreme weakness, myalgias, thirst, dry mouth, polyuria. BP is 140/95 mm Hg, heart rate is 98/min., no edemas or smell of acetone detected. • What measures should be taken to prevent development of comatose state in the patient? A. Stop metformin, prescribe short-acting insulin B. Double the dosage of metformin C. Apply hypotonic solution of sodium chloride D. Additionally prescribe long-acting insulin E. Prescribe glibenclamid
  • 24. Step • 51. A 27-year-old nurse comes to the emergency department because of nervousness, dizziness, palpitations, and excess perspiration for the past 3 hours. She has had similar episodes over the past 6 months. The symptoms improve following ingestion of orange juice or soft drinks. She says that she has had a great deal of stress. She has been drinking two alcoholic beverages daily for the past month; before this time, she seldom drank alcohol. Examination shows no abnormalities. Her serum glucose concentration is 30 mg/dL. Intravenous glucose is administered, and the patient's symptoms improve. • Which of the following is the most appropriate next step in diagnosis? A. Liver tests B. Measurement of serum proinsulin and insulin antibodies C. Measurement of serum cortisol and ACTH concentrations D. Measurement of serum growth hormone and plasma somatomedin-C concentrations E. Measurement of serum insulin and C-peptide concentrations
  • 25. • 51. A 27-year-old nurse comes to the emergency department because of nervousness, dizziness, palpitations, and excess perspiration for the past 3 hours. She has had similar episodes over the past 6 months. The symptoms improve following ingestion of orange juice or soft drinks. She says that she has had a great deal of stress. She has been drinking two alcoholic beverages daily for the past month; before this time, she seldom drank alcohol. Examination shows no abnormalities. Her serum glucose concentration is 30 mg/dL. Intravenous glucose is administered, and the patient's symptoms improve. • Which of the following is the most appropriate next step in diagnosis? A. Liver tests B. Measurement of serum proinsulin and insulin antibodies C. Measurement of serum cortisol and ACTH concentrations D. Measurement of serum growth hormone and plasma somatomedin-C concentrations E. Measurement of serum insulin and C-peptide concentrations
  • 26. • 54. Three days after hospitalization for diabetic ketoacidosis, an 87-year- old woman refuses insulin injections. She says that her medical condition has declined so much that she no longer wishes to go on living; she is nearly blind and will likely require bilateral leg amputations. She reports that she has always been an active person and does not see how her life will be of value anymore. She has no family and most of her friends are sick or deceased. On mental status examination, she is alert and cooperative. She accurately describes her medical history and understands the consequences of refusing insulin. There is no evidence of depression. She dismisses any attempts by the physician to change her mind, saying that the physician is too young to understand her situation. She says, "I know I will die, and this is what I want." • Which of the following is the most appropriate next step in management? A. Discharge the patient after she has signed an "against medical advice" form B. Seek a court order to appoint a legal guardian C. Offer insulin but allow the patient to refuse it D. Admit to the psychiatric unit E. Administer insulin against the patient's wishes
  • 27. • 54. Three days after hospitalization for diabetic ketoacidosis, an 87-year- old woman refuses insulin injections. She says that her medical condition has declined so much that she no longer wishes to go on living; she is nearly blind and will likely require bilateral leg amputations. She reports that she has always been an active person and does not see how her life will be of value anymore. She has no family and most of her friends are sick or deceased. On mental status examination, she is alert and cooperative. She accurately describes her medical history and understands the consequences of refusing insulin. There is no evidence of depression. She dismisses any attempts by the physician to change her mind, saying that the physician is too young to understand her situation. She says, "I know I will die, and this is what I want." • Which of the following is the most appropriate next step in management? A. Discharge the patient after she has signed an "against medical advice" form B. Seek a court order to appoint a legal guardian C. Offer insulin but allow the patient to refuse it D. Admit to the psychiatric unit E. Administer insulin against the patient's wishes
  • 28. • A previously healthy 18-year-old man is brought to the emergency department because of abdominal pain and nausea for 6 hours. He has had decreased appetite for the past week. He takes no medications. He drinks one to two beers daily and occasionally more on weekends. He does not use illicit drugs. His temperature is 37.8°C (100°F), pulse is 120/min, respirations are 24/min, and blood pressure is 105/60 mm Hg. Abdominal examination shows diffuse tenderness with no guarding or rebound. Bowel sounds are normal. Laboratory studies show: • Serum Na+ 135 mEq/L Cl− 98 mEq/L K+ 3.8 mEq/L HCO3− 16 mEq/L Glucose 360 mg/dL Ketones present Urine ketones present • Arterial blood gas analysis on room air shows a pH of 7.30. Which of the following is the most likely diagnosis? A. Acute appendicitis B. Acute pancreatitis C. Alcoholic ketoacidosis D. Diabetic ketoacidosis E. Lactic acidosis
  • 29. • A previously healthy 18-year-old man is brought to the emergency department because of abdominal pain and nausea for 6 hours. He has had decreased appetite for the past week. He takes no medications. He drinks one to two beers daily and occasionally more on weekends. He does not use illicit drugs. His temperature is 37.8°C (100°F), pulse is 120/min, respirations are 24/min, and blood pressure is 105/60 mm Hg. Abdominal examination shows diffuse tenderness with no guarding or rebound. Bowel sounds are normal. Laboratory studies show: • Serum Na+ 135 mEq/L Cl− 98 mEq/L K+ 3.8 mEq/L HCO3− 16 mEq/L Glucose 360 mg/dL Ketones present Urine ketones present • Arterial blood gas analysis on room air shows a pH of 7.30. Which of the following is the most likely diagnosis? A. Acute appendicitis B. Acute pancreatitis C. Alcoholic ketoacidosis D. Diabetic ketoacidosis E. Lactic acidosis
  • 31. Predisposing factors  Diagnostic criteria Laboratory findings • sudden withdrawal of corticosteroid hormone therapy in patients with chronic insufficiency and non- endocrine disorders; • stress or infection in patints with Adisson’s disease (especially with septicemia, trauma, surgery, prolonged fasting, salt loss due to excessive sweating during hot weather); • thrombosis or embolism of primary affected adrenals (Waterhouse- Friderichsen syndrome). • An adrenal crisis is characterized by profound asthenia, cardiovascular, gastrointestinal, neuro- psychyatric disorders. • Cardiovascular disorders include hypotension, cyanosis, dyspnea. • Hypotension and peripheral vascular collapse is result of gastrointestinal disturbances (nausea, vomiting diarrhea) also can lead to renal shutdown with azotemia. • Body temperature may be subnormal, through severe hyperthermia due to infection is often seen. • electrolyte disturbances: a low serum sodium level and a high serum potassium level • hypoglycemia • decreased cortisol level • ACTH increased in primary and decreased in secondary adrenal insufficiency
  • 32. • A 34 y.o. patient has been suffering from pulmonary tuberculosis for 7 years; he complains of muscle weakness, weight loss, diarrhea, frequent urination. Objectively: hyperpigmentation of skin, gums, internal surface of cheeks. AP- 90/58 mm Hg. Blood count: RBC- 3, 1 ∗ 1012/L, Hb- 95 g/L, C.I.- 0,92; leukocytes - 9, 4 ∗9/L, eosinophils - 7, segmentonuclear leukocytes - 45, stab neutrophils - 1, lymphocytes - 40, monocytes - 7, Na+- 125 mmol/L, K+- 7,3 mmol/L. • What is the preliminary diagnosis? A. Pheochromocytoma B. Primary hyperaldosteronism C. Congenital adrenocortical hyperplasia D. Diabetes insipidus E. Primary adrenocortical insufficiency
  • 33. • A 34 y.o. patient has been suffering from pulmonary tuberculosis for 7 years; he complains of muscle weakness, weight loss, diarrhea, frequent urination. Objectively: hyperpigmentation of skin, gums, internal surface of cheeks. AP- 90/58 mm Hg. Blood count: RBC- 3, 1 ∗ 1012/L, Hb- 95 g/L, C.I.- 0,92; leukocytes - 9, 4 ∗9/L, eosinophils - 7, segmentonuclear leukocytes - 45, stab neutrophils - 1, lymphocytes - 40, monocytes - 7, Na+- 125 mmol/L, K+- 7,3 mmol/L. • What is the preliminary diagnosis? A. Pheochromocytoma B. Primary hyperaldosteronism C. Congenital adrenocortical hyperplasia D. Diabetes insipidus E. Primary adrenocortical insufficiency
  • 34. • After having the flu, a 39-year-old male patient with a history of Addison’s disease developed a condition manifested by weakness, depression, nausea, vomi- ting, diarrhea, hypoglycemia. AP- 75/50 mm Hg. Blood test results: low corticosterone and cortisol, 13- oxycorticosteroids, 17-oxycorticosteroids levels. • What condition developed in the patient? A. Acute adrenal insufficiency B. Acute gastritis C. Acute enterocolitis D. Collapse E. Diabetes mellitus
  • 35. • After having the flu, a 39-year-old male patient with a history of Addison’s disease developed a condition manifested by weakness, depression, nausea, vomi- ting, diarrhea, hypoglycemia. AP- 75/50 mm Hg. Blood test results: low corti-costerone and cortisol, 13- oxycorticosteroids, 17-oxycorticosteroids levels. • What condition developed in the patient? A. Acute adrenal insufficiency B. Acute gastritis C. Acute enterocolitis D. Collapse E. Diabetes mellitus
  • 36. • A 41-year-old patient with Addison’s disease had influenza. After that he developed adynamia, depression, nausea, vomiting, diarrhea and hypoglycemia. BP is 75/50 mm Hg. Blood test: decreased content of corticosterone, hydrocortisone, 13-oxycorticosteroids, 17-oxycorticosteroids. • What condition has developed in the patient? A. Acute adrenal gland insufficiency B. Acute gastritis C. Acute enterocolitis D. Collapse E. Diabetes mellitus
  • 37. • A 41-year-old patient with Addi-son’s disease had influenza. After that he developed adynamia, depression, nausea, vomiting, diarrhea and hypoglycemia. BP is 75/50 mm Hg. Blood test: decreased content of corticosterone, hydrocortisone, 13-oxycorticosteroids, 17-oxycorticosteroids. • What condition has developed in the patient? A. Acute adrenal gland insufficiency B. Acute gastritis C. Acute enterocolitis D. Collapse E. Diabetes mellitus
  • 38. • A 43-year-old female patient was delivered to the hospital in grave condition. She has a history of Addison’s disease. The patient had been regularly taking prednisolone but a week before she stopped taking this drug. Objectively: sopor, skin and visible mucous membranes are pigmented, skin and muscle turgor is decreased. Heart sounds are muffled, rapid. AP- 60/40 mm Hg, heart rate - 96/min. In blood: Na - 120 mi-llimole/l, K - 5,8 millimole/l. • Development of this complication is primarily caused by the deficit of the following hormone: A. Cortisol B. Corticotropin (ACTH) C. Adrenaline D. Noradrenaline E. Adrostendion
  • 39. • A 43-year-old female patient was delivered to the hospital in grave condi-tion. She has a history of Addison’s di-sease. The patient had been regularly taking prednisolone but a week before she stopped taking this drug. Objecti-vely: sopor, skin and visible mucous membranes are pigmented, skin and muscle turgor is decreased. Heart sounds are muffled, rapid. AP- 60/40 mm Hg, heart rate - 96/min. In blood: Na - 120 mi-llimole/l, K - 5,8 millimole/l. • Development of this complication is primarily caused by the deficit of the following hormone: A. Cortisol B. Corticotropin (ACTH) C. Adrenaline D. Noradrenaline E. Adrostendion
  • 41. Etiology  Diagnostic criteria Laboratory findings is unknown Symptoms “The 5 P’s” • Pressure increase (hypertension ) • Palpitation (tachycardia) • Perspiration • Pain (abrupt onset of throbbing headache, chest (angina), abdominal pain) • Pallor (due to vasoconstriction) cold and clammy skin • An increased 3-h (24-h) urinary excretion of epinephrine, norepinephrine and their metabolic products (VMA or metanephrines). • Plasma free metanephrine is up to 99% sensitive. Imaging tests to localize tumor: • CT scanning and MRI of the chest and abdomen with and without contrast. • Radiopharmaceuticals with nuclear imaging techniques can help to localize pheochromocytomas when the lesion is large enough to be obvious on CT or MRI. 123I-metaiodobenzylguanidine (MIBG) is the most used compound for this investigation. Normal adrenal tissue rarely picks up this isotope, but 90% of pheochromocytomas do.
  • 42. • A 32-year-old woman complains of dizziness, headache, palpitation, tremor. For the last several months she has been under outpatient monitoring for increased arterial pressure. Recently such attacks have become more frequent and severe. Objectively: skin is covered with clammy sweat, tremor of the extremities is present. Heart rate - 110/min, BP - 220/140 mm Hg. Heart sounds are weakened. In blood: WBCs - 9, 8 · 109/l, ESR - 22 mm/h. Blood glucose - 9,8 millimole/l. • What disease is the most likely cause of this crisis? A. Pheochromocytoma B. Essential hypertension C. Preeclampsia D. Primary hyperaldosteronism E. Diabetic glomerulosclerosis
  • 43. • A 32-year-old woman complains of dizziness, headache, palpitation, tremor. For the last several months she has been under outpatient monitoring for increased arterial pressure. Recently such attacks have become more frequent and severe. Objectively: skin is covered with clammy sweat, tremor of the extremities is present. Heart rate - 110/min, BP - 220/140 mm Hg. Heart sounds are weakened. In blood: WBCs - 9, 8 · 109/l, ESR - 22 mm/h. Blood glucose - 9,8 millimole/l. • What disease is the most likely cause of this crisis? A. Pheochromocytoma B. Essential hypertension C. Preeclampsia D. Primary hyperaldosteronism E. Diabetic glomerulosclerosis
  • 44. • A 51-year-old woman complains of headache, trembling, paresthesiae, palpitations, increased blood pressure up to 280/160 mm Hg. The day before she experienced exhausting headache, vascular pulsation, palpitations, asphyxia, stomachache, unbearable fear of coming death. The patient paled and broke out in cold sweat. In urine there is increased content of vanillylmandelic acid. • What disease causes such clinical presentation in the patient? A. Pheochromocytoma B. Conn’s syndrome (primary hyperaldosteronism) C. Cushing’s syndrome D. Primary hypertension E. Cushing’s disease
  • 45. • A 51-year-old woman complains of headache, trembling, paresthesiae, palpitations, increased blood pressure up to 280/160 mm Hg. The day before she experienced exhausting headache, vascular pulsation, palpitations, asphyxia, stomachache, unbearable fear of coming death. The patient paled and broke out in cold sweat. In urine there is increased content of vanillylmandelic acid. • What disease causes such clinical presentation in the patient? A. Pheochromocytoma B. Conn’s syndrome (primary hyperaldosteronism) C. Cushing’s syndrome D. Primary hypertension E. Cushing’s disease
  • 46. • A 38 y.o. woman suffers from paroxysmal AP rises up to 240/120 mm Hg accompanied by nausea, vomiting, tachycardia, excessive sweating. During the onset blood is hyperglycemic. After the onset there is voluminous urination. Kidneys sonography revealed accessory mass bordering upon the upper pole of the right kidney; presumably it belongs to the adrenal gland. • What laboratory test will allow making a more precise diagnosis? A. Estimation of catecholamine and vanillyl-mandelic acid excretion with urine B. Estimation of insulin and C-peptide content in blood C. Estimation of glomerular filtration rate D. Estimation of thyroxin and thyrotrophic hormone in blood E. Estimation of rennin content in blood
  • 47. • A 38 y.o. woman suffers from paroxysmal AP rises up to 240/120 mm Hg accompanied by nausea, vomiting, tachycardia, excessive sweating. During the onset blood is hyperglycemic. After the onset there is voluminous urination. Kidneys sonography revealed accessory mass bordering upon the upper pole of the right kidney; presumably it belongs to the adrenal gland. • What laboratory test will allow making a more precise diagnosis? A. Estimation of catecholamine and vanillyl-mandelic acid excretion with urine B. Estimation of insulin and C-peptide content in blood C. Estimation of glomerular filtration rate D. Estimation of thyroxin and thyrotrophic hormone in blood E. Estimation of rennin content in blood
  • 48. • A 49 y.o. female patient was admitted to the hospital with acute attacks of headache accompanied by pulsation in temples, AP rised up to 280/140 mm Hg. Pheochromocytoma is suspected. • What mechanism of hypertensive atack does this patient have? A. Increasing of aldosterone level in blood B. Increasing of plasma renin activity C. Increasing of vasopressin excretion D. Increasing of thyroxine excretion E. Increasing of catecholamines concentration
  • 49. • A 49 y.o. female patient was admitted to the hospital with acute attacks of headache accompanied by pulsation in temples, AP rised up to 280/140 mm Hg. Pheochromocytoma is suspected. • What mechanism of hypertensive atack does this patient have? A. Increasing of aldosterone level in blood B. Increasing of plasma renin activity C. Increasing of vasopressin excretion D. Increasing of thyroxine excretion E. Increasing of catecholamines concentration
  • 50. • A patient with suspected pheochromocytoma has normal blood pressure in the periods between the atacks and a tendency to tachycardia. Urine test revealed no pathology. It was decided to use a provocative test with histamine. • What medication should be prepared to provide emergency care in case of a positive test result? A. Pipolphen B. Phentolamine C. Nifedipine D. Mesatonum E. Prednisolone
  • 51. • A patient with suspected pheochromocytoma has normal blood pressure in the periods between the atacks and a tendency to tachycardia. Urine test revealed no pathology. It was decided to use a provocative test with histamine. • What medication should be prepared to provide emergency care in case of a positive test result? A. Pipolphen B. Nifedipine C. Phentolamine D. Mesatonum E. Prednisolone
  • 52. • A patient has been provisionally diagnosed with pheochromocytoma at the stage of intermission. BP is within norm, there is a tendency towards tachycardia. No urine pathologies. The decision has been made to perform a provocative test with histamine. • What drug should be kept close at hand for emergency aid in case of positive test result? A. Phentolamine B. PipolphenB C. Nifedipine D. Mesaton (Phenylephrine) E. Prednisolone
  • 53. • A patient has been provisionally diagnosed with pheochromocytoma at the stage of intermission. BP is within norm, there is a tendency towards tachycardia. No urine pathologies. The decision has been made to perform a provocative test with histamine. • What drug should be kept close at hand for emergency aid in case of positive test result? A. Phentolamine B. PipolphenB C. Nifedipine D. Mesaton (Phenylephrine) E. Prednisolone
  • 54. • A 40 y.o. patient was diagnosed: 1. Medullar thyroid gland cancer. 2. Pheochromocytoma. • What operation should be performed at first? A. Operation on thyroid gland B. Operation on account of pheochromocytoma C. Krail’s operation D. Subtotal resection of thyroid gland and fascicular resection of lymphatic nodes E. Vanach’s operation
  • 55. • A 40 y.o. patient was diagnosed: 1. Medullar thyroid gland cancer. 2. Pheochromocytoma. • What operation should be performed at first? A. Operation on thyroid gland B. Operation on account of pheochromocytoma C. Krail’s operation D. Subtotal resection of thyroid gland and fascicular resection of lymphatic nodes E. Vanach’s operation
  • 56. Step • 45. A 42-year-old man comes to the office for preoperative evaluation prior to undergoing adrenalectomy scheduled in 2 weeks. One month ago, he received care in the emergency department for pain over his right flank following a motor vehicle collision. At that time, blood pressure was 160/100 mm Hg and CT scan of the abdomen showed an incidental 10-cm left adrenal mass. Results of laboratory studies, including complete blood count, serum electrolyte concentrations, and liver function tests, were within the reference ranges. The patient otherwise had been healthy and had never been told that he had elevated blood pressure. He takes no medications. A follow-up visit in the office 2 weeks ago disclosed elevated urinary normetanephrine and metanephrine and plasma aldosterone concentrations. The patient was referred to a surgeon, who recommended the adrenalectomy. Today, vital signs are temperature 36.6°C (97.9°F), pulse 100/min, respirations 14/min, and blood pressure 170/95 mm Hg. Physical examination discloses no significant findings. Initial preoperative preparation should include treatment with which of the following? A. Labetalol B. A loading dose of potassium chloride C. Nifedipine D. Phenoxybenzamine E. Spironolactone
  • 57. • 45. A 42-year-old man comes to the office for preoperative evaluation prior to undergoing adrenalectomy scheduled in 2 weeks. One month ago, he received care in the emergency department for pain over his right flank following a motor vehicle collision. At that time, blood pressure was 160/100 mm Hg and CT scan of the abdomen showed an incidental 10-cm left adrenal mass. Results of laboratory studies, including complete blood count, serum electrolyte concentrations, and liver function tests, were within the reference ranges. The patient otherwise had been healthy and had never been told that he had elevated blood pressure. He takes no medications. A follow-up visit in the office 2 weeks ago disclosed elevated urinary normetanephrine and metanephrine and plasma aldosterone concentrations. The patient was referred to a surgeon, who recommended the adrenalectomy. Today, vital signs are temperature 36.6°C (97.9°F), pulse 100/min, respirations 14/min, and blood pressure 170/95 mm Hg. Physical examination discloses no significant findings. Initial preoperative preparation should include treatment with which of the following? A. Labetalol B. A loading dose of potassium chloride C. Nifedipine D. Phenoxybenzamine E. Spironolactone
  • 58. • 23. A 30-year-old woman comes to the physician because of intermittent throbbing headaches, sweating, and pallor over the past 3 months. She has had several blood pressure measurements that fluctuate from 110/80 mm Hg to 160/108 mm Hg. Her pulse is 100/min, and blood pressure now is 138/88 mm Hg. • Serum studies show: Na+ 140 mEq/L Cl− 110 mEq/L K + 4.5 mEq/L HCO3 − 26 mEq/L Urea nitrogen 14 mg/dL Creatinine 1 mg/dL • Which of the following is the most likely location of the abnormality? A. Adrenal cortex B. Adrenal medulla C. Aorta D. Renal arterioles E. Renal glomeruli F. Thyroid gland
  • 59. • 23. A 30-year-old woman comes to the physician because of intermittent throbbing headaches, sweating, and pallor over the past 3 months. She has had several blood pressure measurements that fluctuate from 110/80 mm Hg to 160/108 mm Hg. Her pulse is 100/min, and blood pressure now is 138/88 mm Hg. • Serum studies show: Na+ 140 mEq/L Cl− 110 mEq/L K + 4.5 mEq/L HCO3 − 26 mEq/L Urea nitrogen 14 mg/dL Creatinine 1 mg/dL • Which of the following is the most likely location of the abnormality? A. Adrenal cortex B. Adrenal medulla C. Aorta D. Renal arterioles E. Renal glomeruli F. Thyroid gland
  • 61. Predisposing factors  Diagnostic criteria  seen shortly after thyroidectomy or 131I therapy in an inadequately prepared patient;  severe infection in patients with previously unrecognized or inadequately treated hyperthyroidism;  Trauma; surgery;  embolism, cerebrovascular accidents or acute cirinary syndrome;  diabetic acidosis;  toxemia of pregnancy; labor;  withdrawal of antithyroid medication;  radiation thyroiditis. • fever; • dehydration; • extreme restlessness with wide emotional swings; • confusion; • delirium, psychosis or even coma; • hepatomegaly with mild jaundice; • tachycardia, atrial fibrillation • marked weakness and muscle wasting; cardiac failure, • the patient may present with cardiovascular collapse or shock
  • 62. • On the first day after a surgery for diffuse toxic goiter a patient developed difficulty breathing, cold sweats, weakness. Objectively: pale skin, body temperature - 38, 5oC , RR - 25/min, Ps- 110/min, AP-90/60 mm Hg. • What early postoperative complication occurred in the patient? A. Thyrotoxic crisis B. Hypothyroid crisis C. Postoperative tetany D. Acute thyroiditis E. Compression of the trachea by the hematoma
  • 63. • On the first day after a surgery for diffuse toxic goiter a patient developed di-fficulty breathing, cold sweats, weakness. Objectively: pale skin, body temperature - 38, 5oC , RR - 25/min, Ps- 110/min, AP-90/60 mm Hg. What early postoperative complication occurred in the patient? A. Thyrotoxic crisis B. Hypothyroid crisis C. Postoperative tetany D. Acute thyroiditis E. Compression of the trachea by the hematoma
  • 65. Predisposing factors  Diagnostic criteria • exposure to cold; • acute infection; • trauma; • myocardial infarction, cardiac failure; • drugs (sedatives, barbiturates, amiodarone, anesthetic agents, diuretics, beta-blockers); • hyponatremia, hypoxia, hypercapnea. • extreme hypothermia (temperatures 24 to 320C), areflexia, seizures, CO2 retention, and respiratory depression caused by decreased cerebral blood flow. Severe hypothermia may be missed unless special low reading thermometers are used. Rapid diagnosis (based on clinical judgement, history, and physical examination) is imperative because early death is likely. • Neuropsychiatric manifestation: develops psychosis with delusions and hallucinations (“myxedema madness”), progressing to depressed level of consciousness, convulsions and coma.
  • 66. • A 39-year-old female patient complains of rapid fatigability, drowsiness, dry skin, hair loss, swelling of the face. A month ago, she underwent a surgery for thyrotoxicosis. • The patient has the following gland dysfunction: A. Thyroid (hypothyroidism), due to inadequate operative technique B. Pituitary, due to a tumor C. Adrenal D. Parathyroid, due to the gland removal during surgery E. Ovarian, due to a tumor
  • 67. • A 39-year-old female patient complains of rapid fatigability, drowsiness, dry skin, hair loss, swelling of the face. A month ago, she underwent a surgery for thyrotoxicosis. • The patient has the following gland dysfunction: A. Thyroid (hypothyroidism), due to inadequate operative technique B. Pituitary, due to a tumor C. Adrenal D. Parathyroid, due to the gland removal during surgery E. Ovarian, due to a tumor
  • 68. Step • 62. A 62-year-old woman comes to the physician for a routine health maintenance examination. On questioning, she has had fatigue, constipation, and a 9-kg (20-lb) weight gain during the past year. She receives estrogen replacement therapy. Serum lipid studies were within the reference range 5 years ago. She is 157 cm (5 ft 2 in) tall and weighs 77 kg (170 lb); BMI is 31 kg/m2 . Physical examination shows no other abnormalities. • Serum lipid studies today show: Total cholesterol 269 mg/dL HDL- cholesterol 48 mg/dL LDL-cholesterol 185 mg/dL Triglycerides 180 mg/dL • Which of the following is the most likely cause? A. Alcohol B. Diabetes mellitus C. Estrogen deficiency D. Estrogen replacement therapy E. Hypothyroidism F. Thiazide diuretic therapy
  • 69. • 62. A 62-year-old woman comes to the physician for a routine health maintenance examination. On questioning, she has had fatigue, constipation, and a 9-kg (20-lb) weight gain during the past year. She receives estrogen replacement therapy. Serum lipid studies were within the reference range 5 years ago. She is 157 cm (5 ft 2 in) tall and weighs 77 kg (170 lb); BMI is 31 kg/m2 . Physical examination shows no other abnormalities. • Serum lipid studies today show: Total cholesterol 269 mg/dL HDL- cholesterol 48 mg/dL LDL-cholesterol 185 mg/dL Triglycerides 180 mg/dL • Which of the following is the most likely cause? A. Alcohol B. Diabetes mellitus C. Estrogen deficiency D. Estrogen replacement therapy E. Hypothyroidism F. Thiazide diuretic therapy
  • 71. Etiology Clinical signs Laboratory findings • Postoperative: due to damage to the thyroid gland during thyroid surgery (commonest cause); • Idiopathic • Pseudohypoparathyroidi sm (resistance to PTH) • Acute severe, hypocalcemia may cause laryngospasm, confusion, seizures, vascular collapse with bradycardia and decompensated heart failure. • Acute, moderate hypocalcemia may cause increased excitability of nerves and muscules, leading to circumoral or distal parasthesia and tetany. • Positive Chvostek’s and Trousseau’s signs • serum calcium is low • serumPTH is low (or inappropriatery normal in the setting of hypocalcemia is indicative of hypoparathyroidism: a high levelof PTH is often found in patients with vitamin D deficiency, PTH resistance, and hyperphosphatemia); • serum phosphate is high
  • 72. • A 63 y.o. patient was operated on account of big multinodular euthyroid goiter. Despite of techical difficulties a forced subtotal resection of both parts of the thyroid gland was performed. On the 4-th day after the operation the woman had cramps of face muscles and upper extremities, stomach ache. Positive Chvostek’s and Trousseau’s signs. • What is the most probable cause of such condition? A. Insufficiency of parathyroid glands B. Postoperative hypothyroidism C. Thyrotoxic crisis D. Injury of recurrent nerve E. Tracheomalacia
  • 73. • A 63 y.o. patient was operated on account of big multinodular euthyroid goiter. Despite of techical difficulties a forced subtotal resection of both parts of the thyroid gland was performed. On the 4-th day after the operation the woman had cramps of face muscles and upper extremities, stomach ache. Positive Chvostek’s and Trousseau’s signs. • What is the most probable cause of such condition? A. Insufficiency of parathyroid glands B. Postoperative hypothyroidism C. Thyrotoxic crisis D. Injury of recurrent nerve E. Tracheomalacia
  • 74. Step • 36. 60-year-old man had a total thyroidectomy and excision of enlarged left jugular lymph nodes for follicular carcinoma. The operation was uncomplicated. He is receiving intravenous 5% dextrose and 0.45% saline with potassium. Twelve hours after the operation he develops circumoral numbness and paresthesias in his fingertips, and he becomes very anxious. Vital signs are temperature 37.6°C (99.7°F), pulse 90/min, respirations 16/min, and blood pressure 140/90 mm Hg. Physical examination discloses a dry neck dressing and no stridor. Extremities are warm, with brisk capillary refill time. • Additional physical examination is most likely to show which of the following? A. Babinski sign present bilaterally B. Chvostek sign C. Deviation of the tongue to the left side D. A drooping left shoulder E. Hyporeflexia
  • 75. • 36. 60-year-old man had a total thyroidectomy and excision of enlarged left jugular lymph nodes for follicular carcinoma. The operation was uncomplicated. He is receiving intravenous 5% dextrose and 0.45% saline with potassium. Twelve hours after the operation he develops circumoral numbness and paresthesias in his fingertips, and he becomes very anxious. Vital signs are temperature 37.6°C (99.7°F), pulse 90/min, respirations 16/min, and blood pressure 140/90 mm Hg. Physical examination discloses a dry neck dressing and no stridor. Extremities are warm, with brisk capillary refill time. • Additional physical examination is most likely to show which of the following? A. Babinski sign present bilaterally B. Chvostek sign C. Deviation of the tongue to the left side D. A drooping left shoulder E. Hyporeflexia
  • 76. Anurea Oliguria - Urine output in the amount of <500 ml per day Anuria - Urine output in the amount of <50 ml per day
  • 77. The main reasons for AKI Absolute indications towards for renal replacement therapy in AKD 1. Inadequate perfusion of kidneys (shock, collapse, acute bleeding, renal artery thrombosis, etc.). 2. Inflammatory and nephrotoxic factors (infections, poisoning with mushrooms, heavy metals salts, chloric hydrocarbons, etc.). 3. Nephrotoxic drugs (drugs, X- ray contrast agents). 4. Obstruction (occlusion) of urinary tracts (tumors, urolithiasis, strictures, ligatures). • Anuria(≤100 ml/24 hr or ≤50 ml/12 hr). • Decompensated metabolic acidosis (рН <7.1 mmol/L). • Azotemia (blood urea >30 mmol/L or blood creatinine >350 µmol/L). • Dysnatriemia (115> Na+ >160 mmol/L). • Hyperpotassemia (К+>6.5 mmol/L). • Hypermagnesemia 4 mmol/L and more in patients with anuria and absence of deep tendon reflexes. • Hyperhydratation with diuretic resistance or the risk of pulmonary edema and/or the brain edema. • Overdose of dialysate drugs. • Coagulopathy, which requires infusion of a large number of blood products in a patient with a risk of pulmonary edema or acute respiratory distress syndrome. • Lactate acidosis as a result of metformin
  • 78. • 2 weeks after recovering from tonsillitis an 8-year-old boy developed edemas of face and lower limbs. Objectively: the patient is in grave condition, BP - 120/80 mm Hg. Urine is of dark brown colour. Oliguria is present. On urine analysis: specific gravity - 1,015, protein - 1,2 g/l, RBCs are leached and cover the whole vision field, granular casts - 1-2 in the vision field, salts are represented by urates (large quantity). • What is the most likely diagnosis? A. Acute glomerulonephritis with nephritic syndrome B. Acute glomerulonephritis with nephrotic syndrome C. Acute glomerulonephritis with nephrotic syndrome, hematuria and hypertension D. Acute glomerulonephritis with isolated urinary syndrome E. Nephrolithiasis
  • 79. • 2 weeks after recovering from tonsillitis an 8-year-old boy developed edemas of face and lower limbs. Objectively: the patient is in grave condition, BP - 120/80 mm Hg. Urine is of dark brown colour. Oliguria is present. On urine analysis: specific gravity - 1,015, protein - 1,2 g/l, RBCs are leached and cover the whole vision field, granular casts - 1-2 in the vision field, salts are represented by urates (large quantity). • What is the most likely diagnosis? A. Acute glomerulonephritis with nephritic syndrome B. Acute glomerulonephritis with nephrotic syndrome C. Acute glomerulonephritis with nephrotic syndrome, hematuria and hypertension D. Acute glomerulonephritis with isolated urinary syndrome E. Nephrolithiasis
  • 80. • A 50 y.o. woman who suffers from chronic pyelonephritis was prescribed a combination of antibiotics for the period of exacerbation - gentamicin (80 mg 3 times a day) and biseptol (960 mg twice a day). • What consequences may be caused by such a combination of antibiotics? A. Glomerulosclerosis B. Acute renal insufficiency C. Chronic renal insufficiency D. Antibiotic combination is optimal and absolutely safe E. Acute suprarenal insufficiency
  • 81. • A 50 y.o. woman who suffers from chronic pyelonephritis was prescribed a combination of antibiotics for the period of exacerbation - gentamicin (80 mg 3 times a day) and biseptol (960 mg twice a day). • What consequences may be caused by such a combination of antibiotics? A. Glomerulosclerosis B. Acute renal insufficiency C. Chronic renal insufficiency D. Antibiotic combination is optimal and absolutely safe E. Acute suprarenal insufficiency
  • 82. • A 33 year old patient has acute blood loss (erythrocytes - 2,2·1012/l, Hb- 55 g/l), blood group is A(II)Rh+. Accidentally the patient go tо transfusion of donor pack edred blood cells of AB(IV) Rh+group. An hour later the patient became anxious, got abdominal andlumbar pain. Ps- 134 bpm, AP- 100/65 mm Hg, body temperature - 38,6oC. After catheterization of urinary bladder 12 ml/h of dark-brown urine were obtained. • What complication is it? A. Cardiac shock B. Acute renal insufficiency C. Allergic reaction to the donor red blood cells D. Citrate intoxication E. Toxic infectious shock
  • 83. • A 33 year old patient has acute blood loss (erythrocytes - 2,2·1012/l, Hb- 55 g/l), blood group is A(II)Rh+. Accidentally the patient go tо transfusion of donor pack edred blood cells of AB(IV) Rh+group. An hour later the patient became anxious, got abdominal andlumbar pain. Ps- 134 bpm, AP- 100/65 mm Hg, body temperature - 38,6oC. After catheterization of urinary bladder 12 ml/h of dark-brown urine were obtained. • What complication is it? A. Cardiac shock B. Acute renal insufficiency C. Allergic reaction to the donor red blood cells D. Citrate intoxication E. Toxic infectious shock
  • 84. • Among the following causes of acute renal failure, the one that would be classified as "postrenal" is: A. Calculi B. Cardiac failure C. Septicemia D. Rhabdomyolysis E. Acute glomerulonephritis
  • 85. • Among the following causes of acute renal failure, the one that would be classified as "postrenal" is: A. Calculi B. Cardiac failure C. Septicemia D. Rhabdomyolysis E. Acute glomerulonephritis
  • 86. • A 24 y.o. patient complains of nausea, vomiting, headache, shortness of breath. He had an acute nephritis being 10 y.o. Proteinuria was found out in urine. Objectively: a skin is grey-pale, the edema is not present. Accent of II tone above aorta. BP 140/100-180/100 mm Hg. Blood level of residual N2- 6,6 mmol/L, creatinine- 406 mmol/L. Day’s diuresis-2300 ml, nocturia. Specific density of urine is 1009, albumin- 0,9 g/L, WBC- 0-2 in f/vis. RBC.- single in f/vis., hyaline casts single in specimen. • Your diagnosis? A. Pheochromocytoma B. Hypertensive illness of the II degree C. Nephrotic syndrome D. Chronic nephritis with violation of kidney function E. Stenosis of kidney artery
  • 87. • A 24 y.o. patient complains of nausea, vomiting, headache, shortness of breath. He had an acute nephritis being 10 y.o. Proteinuria was found out in urine. Objectively: a skin is grey-pale, the edema is not present. Accent of II tone above aorta. BP 140/100-180/100 mm Hg. Blood level of residual N2- 6,6 mmol/L, creatinine- 406 mmol/L. Day’s diuresis-2300 ml, nocturia. Specific density of urine is 1009, albumin- 0,9 g/L, WBC- 0-2 in f/vis. RBC.- single in f/vis., hyaline casts single in specimen. • Your diagnosis? A. Pheochromocytoma B. Hypertensive illness of the II degree C. Nephrotic syndrome D. Chronic nephritis with violation of kidney function E. Stenosis of kidney artery
  • 88. • A 39-year-old man complains of morning headaches, appetite loss, nausea, morning vomiting, periodic nasal hemorrhages. The patient had a case of acute glomerulonephritis at the age of 15. Examination revealed rise of arterial pressure up to 220/130 mm Hg, skin hemorrhages on his arms and legs, pallor of skin and mucous membranes. • What biochemical parameter is the most important for making diagnosis in this case? A. Blood creatinine* B. Blood bilirubin C. Blood sodium D. Uric acid E. Fibrinoge
  • 89. • A 39-year-old man complains of morning headaches, appetite loss, nausea, morning vomiting, periodic nasal hemorrhages. The patient had a case of acute glomerulonephritis at the age of 15. Examination revealed rise of arterial pressure up to 220/130 mm Hg, skin hemorrhages on his arms and legs, pallor of skin and mucous membranes. • What biochemical parameter is the most important for making diagnosis in this case? A. Blood creatinine* B. Blood bilirubin C. Blood sodium D. Uric acid E. Fibrinoge
  • 90. • A 52 y.o. patient fell from 3 m hight on the ground with the right lumbar area. He complains of pain here. There is microhematuria in the urine. On urography kidney’s functioning is satisfactory. • What is the most probable diagnosis? A. Subcapsular kidney’s rupture B. Multiple kidney’s ruptures C. Paranephral hematoma D. Kidney’s contusion E. Kidney’s abruption
  • 91. • A 52 y.o. patient fell from 3 m hight on the ground with the right lumbar area. He complains of pain here. There is microhematuria in the urea. On urography kidney’s functioning is satisfactory. • What is the most probable diagnosis? A. Subcapsular kidney’s rupture B. Multiple kidney’s ruptures C. Paranephral hematoma D. Kidney’s contusion E. Kidney’s abruption
  • 92. • A 35 y.o. male patient suffers from chronic glomerulohephritis and has been on hemodialysis for the last 3 years. He has developed irregularity in the heart activity, hypotension, progressive weakness, dyspnea. On ECG: bradycardia, 1st degree atrioventicular block, high sharpened T-waves. Before he had severely disturbed the drinking and diet regimen. • What is the most likely cause of these changes? A. Hyperkalemia B. Hyperhydratation C. Hypokaliemia D. Hypernatremia E. Hypocalcaemia
  • 93. • A 35 y.o. male patient suffers from chronic glomerulohephritis and has been on hemodialysis for the last 3 years. He has developed irregularity in the heart activity, hypotension, progressive weakness, dyspnea. On ECG: bradycardia, 1st degree atrioventicular block, high sharpened T-waves. Before he had severely disturbed the drinking and diet regimen. • What is the most likely cause of these changes? A. Hyperkalemia B. Hyperhydratation C. Hypokaliemia D. Hypernatremia E. Hypocalcaemia
  • 94. • A 3-year-old girl is being treated at a resuscitation unit with diagnosis ”acute kidney failure, oligoanuric stage”. ECG: high T wave, extended QRS complex, displacement of S-T interval downwards below the isoline. • What electrolyte imbalance is it? A. Hyperkalemia* B. Hypokalemia C. Hypocalcemia D. Hypercalcemia E. Hyperphosphatemia
  • 95. • A 3-year-old girl is being treated at a resuscitation unit with diagnosis ”acute kidney failure, oligoanuric stage”. ECG: high T wave, extended QRS complex, displacement of S-T interval downwards below the isoline. • What electrolyte imbalance is it? A. Hyperkalemia B. Hypokalemia C. Hypocalcemia D. Hypercalcemia E. Hyperphosphatemia
  • 96. • A 41-year-old male patient was delivered to a hospital unconscious. During the previous 7 days he had been taking large doses of biseptolum for a cold. The night before, he began complaining of dyspnea, especially when lying down, swollen legs, 2-day urinary retention. In the morning he had seizures and lost consciousness. Objctively: noisy breathing at the rate of 30/min, edematous legs and lumbar region, Ps- 50/min. Plasma creatinine is 0,586 mmol/l, plasma potassium - 7,2 mmol/l. • What treatment is necessary for this patient? A. Hemodialysis B. Large doses of verospiron C. Plasma volume expanders D. Glucocorticosteroids E. Heparin
  • 97. • A 41-year-old male patient was delivered to a hospital unconscious. During the previ-ous 7 days he had been taking large doses of biseptolum for a cold. The night before, he began complaining of dyspnea, especially when lying down, swollen legs, 2-day urinary retention. In the morning he had seizures and lost consciousness. Objctively: noisy breathi-ng at the rate of 30/min, edematous legs and lumbar region, Ps- 50/min. Plasma creatinine is 0,586 mmol/l, plasma potassium - 7,2 mmol/l. • What treatment is necessary for this patient? A. Hemodialysis B. Large doses of verospiron C. Plasma volume expanders D. Glucocorticosteroids E. Heparin
  • 98. • A 50-year-old male in a grave condition has been admitted to the intensive care unit. It is known from life history that the patient works in agriculture, and 3 hours ago was engaged into insecticide treatment of crops for control of colorado potato beetle. Condition on admission: acrocyanosis, bronchorrhea, tachypnea, AP- 100/60 mm Hg, Ps- 44/min. • What method of efferent therapy would be most appropriate at this stage? A. Hemosorbtion B. Hemodialysis C. Plasmapheresis D. Lymphosorption E. Plasma dialysis
  • 99. • A 50-year-old male in a grave condition has been admitted to the intensive care unit. It is known from life history that the pati-ent works in agriculture, and 3 hours ago was engaged into insecticide treatment of crops for control of colorado potato beetle. Condition on admission: acrocyanosis, bronchorrhea, tachypnea, AP- 100/60 mm Hg, Ps- 44/min. • What method of efferent therapy would be most appropriate at this stage? A. Hemosorbtion B. Hemodialysis C. Plasmapheresis D. Lymphosorption E. Plasma dialysis
  • 100. • A resuscitation unit received a 46-year-old woman, who has been suffering from diabetes mellitus type 1 for approximately 30 years. Objectively: the skin is pale, heart sounds are weakened, BP is 170/100 mm Hg, lower limbs are markedly swollen. Blood creatinine -1125 mcmol/l, urea - 49,6 mmol/l, potassium - 6.3 mmol/l, glucose - 7,6 mmol/l, glomerular filtration rate - 5 ml/min. • What treatment is indicated for the patient in the first place? A. Hemodialysis B. Kidney transplantation C. Hemofiltration D. Enterosorption E. Conservative detoxification therapy
  • 101. • A resuscitation unit received a 46-year-old woman, who has been suffering from diabetes mellitus type 1 for approximately 30 years. Objectively: the skin is pale, heart sounds are weakened, BP is 170/100 mm Hg, lower limbs are markedly swollen. Blood creatinine -1125 mcmol/l, urea - 49,6 mmol/l, potassium - 6.3 mmol/l, glucose - 7,6 mmol/l, glomerular filtration rate - 5 ml/min. • What treatment is indicated for the patient in the first place? A. Hemodialysis B. Kidney transplantation C. Hemofiltration D. Enterosorption E. Conservative detoxification therapy
  • 102. • A 30-year-old woman with a long history of chronic pyelonephritis complains of considerable weakness, sleepiness, decrease in diuresis down to 100 ml per day. BP is 200/120 mm Hg. In blood: creatinine - 0,62 millimole/l, hypoproteinemia, albumines - 32 g/l, potassium - 6,8 millimole/l, hypochromic anemia, increased ESR. • What is the first step in the patient treatment tactics? A. Haemodialysis B. Antibacterial therapy C. Enterosorption D. Haemosorption E. Blood transfusion
  • 103. • A 30-year-old woman with a long history of chronic pyelonephritis complains of consi-derable weakness, sleepiness, decrease in diuresis down to 100 ml per day. BP is 200/120 mm Hg. In blood: creatinine - 0,62 millimole/l, hypoproteinemia, albumines - 32 g/l, potassium - 6,8 millimole/l, hypochromic anemia, increased ESR. • What is the first step in the patient treatment tactics? A. Haemodialysis B. Antibacterial therapy C. Enterosorption D. Haemosorption E. Blood transfusion
  • 104. • A 23-year-old patient after intake of brake fluid has developed anuria that has been lasting for 5 days already. Creatinine level increased up to 0,769 mmol/l. • What treatment tactics should be chosen in the given case? A. Hemodialysis B. Detoxification therapy C. Antidotal therapy D. Diuretics E. Plasmapheresis
  • 105. • A 23-year-old patient after intake of brake fluid has developed anuria that has been lasting for 5 days already. Creatinine level increased up to 0,769 mmol/l. • What treatment tactics should be chosen in the given case? A. Hemodialysis B. Detoxification therapy C. Antidotal therapy D. Diuretics E. Plasmapheresis
  • 106. • A 35-year-old patient has been in the intensive care unit for acute renal failure due to crush for 4 days. Objectively: the patient is inadequate. Breathing rate - 32/min. Over the last 3 hours individual moist rales can be auscultated in lungs. ECG shows high T waves, right ventricular extrasystoles. CVP - 159 mm Hg. In blood: the residual nitrogen - 62 mi-llimole/l, K+- 7,1 millimole/l, C l−- 78 mi-llimole/l, N a+- 120 millimole/l, Ht - 0,32, Hb - 100 g/l, blood creatinine - 0,9 milli- mole/l. • The most appropriate method of treatment would be: A. Hemodialysis B. Plasma sorption C. Hemosorption D. Plasma filtration E. Ultrafiltration
  • 107. • A 35-year-old patient has been in the intensive care unit for acute renal failure due to crush for 4 days. Objectively: the patient is inadequate. Breathing rate - 32/min. Over the last 3 hours individual moist rales can be auscultated in lungs. ECG shows high T waves, right ventri-cular extrasystoles. CVP - 159 mm Hg. In blood: the residual nitrogen - 62 mi- llimole/l, K+- 7,1 millimole/l, C l−- 78 mi-llimole/l, N a+- 120 millimole/l, Ht - 0,32, Hb - 100 g/l, blood creatinine - 0,9 milli-mole/l. • The most appropriate method of treatment would be: A. Hemodialysis B. Plasma sorption C. Hemosorption D. Plasma filtration E. Ultrafiltration
  • 108. • A 30-year-old woman suffers from polycystic renal disease. She has been admitted with signs of fatigue, thirst and nocturia. Diuresis is up to 1800 ml per day. BP is 200/100 mm Hg. Blood test: erythrocytes -1,8 • 109/l, Hb- 68 g/l. Urine analysis: specific gravity - 1005, leukocytes 50-60, erythrocytes - 3-5 in the vision field, creatinine - 0,82 mmol/l, potassium - 6,5 mmol/l, glomerular filtration rate - 10 ml/min. • What tactics would be leading in the patient’s treatment? A. Hemodialysis B. Antibacterial therapy C. Sorbent agents D. Blood transfusion E. Hypotensive therapy
  • 109. • A 30-year-old woman suffers from polycystic renal disease. She has been admitted with signs of fatigue, thirst and nocturia. Diuresis is up to 1800 ml per day. BP is 200/100 mm Hg. Blood test: erythrocytes -1,8 • 109/l, Hb- 68 g/l. Urine analysis: specific gravity - 1005, leukocytes 50-60, erythrocytes - 3-5 in the vision field, creatinine - 0,82 mmol/l, potassium - 6,5 mmol/l, glomerular filtration rate - 10 ml/min. • What tactics would be leading in the patient’s treatment? A. Hemodialysis B. Antibacterial therapy C. Sorbent agents D. Blood transfusion E. Hypotensive therapy
  • 110. • A woman undergoing in-patient treatment for viral hepatitis type B developed headache, nausea, recurrent vomiting, memory lapses, flapping tremor of her hands, rapid pulse. Sweet smell from the mouth is detected. Body temperature is 37,6°C, heart rate is 89/min. • What complication developed in the patient? A. Acute liver failure B. Ischemic stroke C. Gastrointestinal hemorrhage D. Hypoglycemic shock E. Meningoencephalitis
  • 111. • A woman undergoing in-patient treatment for viral hepatitis type B developed headache, nausea, recurrent vomiting, memory lapses, flapping tremor of her hands, rapid pulse. Sweet smell from the mouth is detected. Body temperature is 37,6°C, heart rate is 89/min. • What complication developed in the patient? A. Acute liver failure B. Ischemic stroke C. Gastrointestinal hemorrhage D. Hypoglycemic shock E. Meningoencephalitis
  • 112. Thank You for attention!